Различные способы решения систем уравнений: Разбираемся в линейных уравнениях раз и навсегда

Содержание

Различные способы решения систем двух линейных уравнений с двумя переменными

Тип урока: систематизации и обобщения изученного.

Класс: 9.

Цель урока:

  • Обобщить изученный материал по теме;
  • Проконтролировать степень усвоения знаний и умений по изученной теме;
  • Показать рациональность применения различных способов решения для конкретной системы;
  • Развивать коммуникативные навыки.

Ход урока

Организационный момент (2 мин.)

Работа проходит в группах по 5-6 человек, всего класс разделен на 5 групп.

Актуализация знаний (10 мин.)

Учитель: Мы с вами продолжаем заниматься решением систем двух линейных уравнений с двумя переменными различными способами.

  • Давайте вспомним, что называется системой?
  • Что означает решить систему уравнений?
  • Сколько решений может иметь система уравнений?
  • Как называется система уравнений, имеющая хотя бы одно решение? Не имеющая решений?
  • Что является графиком линейной функции?
  • Система состоит из двух линейных уравнений. А каким может быть взаимное расположение двух прямых на плоскости?
  • Сколько общих точек имеют прямые в каждом из этих случаев?
  • Сколько решений может иметь система в зависимости от взаимного расположения прямых на плоскости?
  • Как узнать, совместна ли система, т.е. имеет ли она решение?
  • Какие способы для нахождения решения системы линейных уравнений существуют?

Учащиеся называют способы решения, на доске появляется следующие записи:

  • способ сложения;
  • способ подстановки;
  • способ сравнения;
  • графический способ;
  • формулы Крамера.

Практическое применение всех способов решения (10мин.)

Для решения систем линейных уравнений нам известно пять способов решения. Предлагаю применить каждый из этих способов для решения следующей системы

.

Выберем самый рациональный способ для данной системы, но не забудем отметить достоинства остальных методов.

У доски по одному представителю от каждой группы решают заданную систему своим способом, который определен с помощью жеребьевки (отрывают лепестки от ромашки, на которых написаны названия способов решения).

Обсуждение решения (3мин)

Подведем итог нашей работы (этап рефлексии).

  • Понравилось ли вам решать систему предложенным вам способом?
  • Что вы чувствовали во время решения?
  • Было ли вам комфортно?
  • Какой из данных методов наиболее оптимален для данной системы уравнений?
  • Во всех ли случаях решения мы получили одинаковый ответ?
  • Почему это произошло?

Практическая работа по выбору рационального способа решения системы линейных уравнений (15 мин)

Однако нельзя хвалить один способ и использовать его при решении всех систем уравнений, а другой считать ненужным, неправильным. Если математики придумали столько различных способов решения, и все они до сих пор применяются на практике, то можно сделать вывод, что для любой системы найдется наиболее рациональный способ. И, наша с вами задача научиться делать этот выбор.

Предлагаю вашему вниманию пять систем. Вам необходимо для каждой из них определить рациональный способ решения и обосновать свой выбор.

Можно предложить следующее соответствие между заданными системами уравнений и способами решения:

а) Способ подстановки;
б) Графический способ;
в) способ сравнения;
г) формулы Крамера;
д) способ сложения.

Учащимся предлагается обосновать такой выбор или внести коррективы. Представители от каждой группы у доски решают ту систему, которая им досталась в результате жеребьевки. Остальные обучающиеся работают на местах.

Подведение итогов урока, рефлексия (5 мин)

Какой же вывод можно вынести из проделанной работы? (учащиеся предлагают свои варианты) Хочется надеяться, что теперь прежде чем решать систему своим любимым способом задумаетесь “ А может быть другой метод решения более удобен, рационален в данной ситуации?”

Домашнее задание: подобрать по две системы линейных уравнений на каждый способ решения.

Способы решения систем уравнений — презентация онлайн

1. Способы решения систем уравнений

2. Различные способы решения систем уравнений


метод подстановки
метод сложения
метод введения новых переменных
графический метод

3. Метод подстановки

• Одно из уравнений системы преобразуют к виду, в
котором y выражено через х ( или х через y )
• Полученное выражение подставляют вместо y (или
вместо х ) во второе
уравнение.
В результате
получается уравнение с одной переменной
• Находят корни этого уравнения
• Воспользовавшись выражением y через х(или х через y),
находят соответствующие значения х (или y)
Метод сложения
• Преобразовать коэффициенты так,
чтобы коэффициенты при х или у
были противоположными числами
• Сложить получившиеся уравнения
• Решить уравнение с одной переменной

5. Метод введения новых переменных

• Замени одно или два выражения в уравнениях системы
новыми переменными так, чтобы вновь полученные
уравнения стали более простыми.
• Реши полученную систему уравнений
методам
наиболее подходящим для э той системы уравнений.
• Сделай обратную замену, для того, чтобы найти
значения первоначальных переменных.
• Запиши ответ в виде пар значений (x,y), которые были
найдены на третьем шаге.

6. Графический метод


Выразить в обоих уравнениях системы
переменную у через переменную х
Построить графики функций в одной
системе координат.
Отметить точки пересечения графиков,
выписать их координаты.
Записать в ответ полученные пары
чисел (х;у).

7. Способ подстановки

Графики пересекаются
в четырех точках (они
обозначены буквами А,
В, С, Д), следовательно,
данная система
уравнений имеет четыре
решения:
(3;-1), (-3;1), (-1;3), (1;-3).
Ответ: (3;-1), (-3;1), (-1;3), (1;-3).
Преимущества и недостатки
метода
Графический метод решения систем, как и графический
метод решения уравнений, красив, но ненадежен:
• во-первых, потому, что графики уравнений мы сумеем
построить далеко не всегда;
• во-вторых, даже если графики уравнений удалось
построить, точки пересечения могут быть не такими
«хорошими», как в специально подобранных примерах
учебника, а то и вовсе могут оказаться за пределами
чертежа.

9. Метод сложения

Проверь себя
Реши систему уравнений, используя метод
x y 5,
подстановки:
xy 6.
Реши систему уравнений, используя метод
3x 2 y 5,
сложения:
2 x 5 y 16.
Реши систему уравнений , используя
графический способ: x y 1,
x y 25.
Реши систему уравнений, используя метод
подстановки: 2 x y 1,
2
2
2
2 x xy y 1.
2

10. Преимущества и недостатки метода

Вывод
Мы рассмотрели четыре различных способов
решения систем уравнений. Каждый выберет для себя
способ, который ему больше всего понравился, самое
главное — что каждый из Вас научился решать
системы такого вида и поэтому эпиграфом могли
служить слова Б.В.Гнеденко:
«Ничто так не содействует
усвоению предмета, как действие
с ним в разных ситуациях»

Урок «Решение систем уравнений различными способами»

Этапы урока

Задачи этапа

Деятельность учителя

Деятельность учащихся

УУД

1.  Организационный момент

Создать благоприятный психологический настрой на работу

Приветствие, проверка подготовленности к учебному занятию, организация внимания детей.

 

 

Включаются в деловой ритм урока.

 

 

 

 

Личностные: самоопределение.

Коммуникативные: планирование учебного сотрудничества с учителем и сверстниками.

2. Актуализация

Актуализация опорных знаний и способов действий.

Работа по кругу. (Внешний, внутренний круги)

1 вариант встает во внутренний круг, 2 вариант встает во внешний круг напротив друг друга.

Повторяем алгоритмы решения систем уравнений  методом подстановки  и методом алгебраического сложения.

1 вопрос: назовите этапы метода подстановки.

Поворачиваемся налево и перемещаемся на 6 человек, разворачиваемся и отвечаем на 2 вопрос

2 вопрос: Назовите этапы метода алгебраического сложения, если имеются противоположные коэффициенты

Поворачиваемся налево и перемещаемся на 3 человека, разворачиваемся и отвечаем на 3 вопрос

3 вопрос: Назовите этапы метода алгебраического сложения, если нет противоположных коэффициентов

После этого рассаживаемся за парты

Учащиеся передвигаются и повторяют известные им алгоритмы. Повторяют вместе с разными людьми, и поэтому идет объективное повторение материала.  Кроме того они сами говорят и слушают одноклассника.

Коммуникативные: развитие устной научной речи, умение слушать и говорить.

Познавательные: анализ и разделение  алгоритма на два случая (2 и 3 вопрос).

3.Целеполагание и мотивация

Обеспечение мотивации учения детьми, принятие ими целей урока.

На доске записаны две системы линейных уравнений.

К доске выходят 4 человека и пробуют решить системы двумя способами.

Ответить на вопрос:

Какую систему, каким способом легче решить? Где  будет более рациональное решение?

 Какая цель нашего урока сегодня?

Решают системы

 

Делают вывод:

Когда удобнее использовать методы решения систем уравнений

 

Цель урока: Решение систем уравнений разными способами, находить удобный способ решения.

Регулятивные: целеполагание.

Коммуникативные: постановка вопросов.

Познавательные: самостоятельное выделение-формулирование цели урока.

4. Закрепление

Выявление пробелов изученного материала, коррекция выявленных пробелов, обеспечение закрепления в памяти детей знаний и способов действий, которые им необходимы для самостоятельной работы по новому материалу.

 

Учащиеся работают в тетради индивидуально. По одному человеку выходят к доске.

Регулятивные: контроль, оценка, коррекция.

Познавательные: умение структуризировать знания, выбор наиболее эффективных способов решения задач, рефлексия способов и условий действия.

 

5. Организация контроля

Выявление качества и уровня усвоения знаний и способов действий, а также выявление недостатков в знаниях и способах действий, установление причин выявленных недостатков.

Самостоятельная работа в парах

Одна работа на двоих. Необходимо рационально распределить задания друг с другом, чтобы успеть выполнить работу.

1 вариант

 

2 вариант

 

10 минут на выполнение работы.

После окончания выполнения учащиеся меняются работами с соседней партой и осуществляют проверку работы одноклассников.

Оценивается выполнение работы и как оценивается работа одноклассников.

Самостоятельное решение в тетради.

 

 

 

 

Взаимоконтроль работы

Регулятивные: контроль, коррекция, самоконтроль и оценка одноклассника.

Личностные: самоопределение, развитие доверия к своему партнеру

Коммуникативные: управление поведением партнера, контроль, коррекция, оценка действий партнера.

6. Подведение итогов урока.

Дать качественную оценку работы класса и отдельных учащихся

-Чему сегодня научились на уроке?

— Что понравилось на уроке?

—Оценить отдельных учащихся

Правильно выбирать способ решения систем уравнений.

Регулятивные: оценка-осознание уровня и качества усвоения; контроль

7. Информация о домашнем задании

Обеспечение понимания детьми цели, содержания и способов выполнения домашнего задания.

№697, 698 решать различными способами

 

 

8. Рефлексия

Инициировать рефлексию детей по поводу психоэмоционального состояния, мотивации их собственной  деятельности и взаимодействия с учителем и другими детьми в классе.

Перед уходом с урока на листе ватмана (лист рефлексии) нарисовать смайлик

Если все было на уроке понятно, то улыбающийся, если остались непонятые вопросы, то нет.

 

Коммуникативные: умение с достаточной полнотой и точностью выражать свои мысли и эмоции;

Познавательные: рефлексия.

Системы линейных уравнений с двумя переменными. Различные способы решения систем линейных уравнений 7 класс

93. Системы линейных уравнений с двумя переменными. Различные способы решения систем линейных уравнений.

Решим задачу. Сумма двух чисел равна 16, а разность – 2. Найдите эти числа.

Обозначим первое число за х, а второе – за у. Тогда сумму чисел запишем как х+у, а разность – х-у.

Так как сумма чисел равна 16, то запишем первое уравнение х+у = 16. Так как разность чисел равна 2, то запишем второе уравнение х-у = 2. Мы составили два уравнения с двумя неизвестными. Надо найти такое решение, которое обращало бы каждое из этих уравнения в верное равенство. В таких случаях говорят, что надо решить систему уравнений.

Уравнения записывают одно под другим, а рядом – фигурную скобку.

x+y=16x-y=2

Пара значений (9;7) является решением для каждого уравнения. Действительно,

9+7=169-7=2

Решением системы уравнений с двумя переменными называется пара значений переменных, обращающая каждое уравнение системы в верное равенство.

Системы линейных уравнений, имеющие одинаковые решения, называются равносильными. Системы линейных уравнений, которые не имеют решений, также считаются равносильными.

Как же найти решение системы линейных уравнений? Первый способ – графический способ решения системы линейных уравнений. Начертим график первого линейного уравнения и график второго линейного уравнения на одной координатной плоскости. Точка, в которой эти графики пересекаются, и будет решением.

Найдем решение системы линейных уравнений графическим способом. Для этого сначала выразим переменную у из каждого уравнения.

2x+5y=6x-y=-11

5y=6-2xy=x+11

y=6-2x5y=x+11

Построим графики функций y=6-2×5 и y=x+11. Каждый из графиков представляет собой прямую, пересекающуюся с осями координат. Найдем точки пересечения каждого из графиков с осями координат. В каждом случае это будут две точки. Через эти две точки проведем прямую. У нас получатся две пересекающиеся прямые.

y=6-2х5

y=x+11

Графики пересекаются в точке с координатами (-7;3). Подставим и проверим, будут ли эти значения решением для системы уравнений

4=6-2(-7)54=-7+11

Равенства выполняются.

Если графики линейных уравнений пересекаются, система имеет одно решение. Если они параллельны, то система не имеет решений. Если прямые совпадают, то решений бесконечно много.

Рассмотрим другие способы решения системы линейных уравнений.

Второй способ – способ подстановки.

Решим систему уравнений

5х+y=6x-y=18

Выразим у из второго уравнения

5x+y=6y=x-18

И подставим в первое уравнение

5x+(х-18)=6y=x-18

Тогда

6x-18=6y=x-18

6x=24y=x-18

x=4y=х-18

x=4y=-14

Ответ: (4;-14).

Третий способ – способ сложения.

Решим систему уравнений

2x-5y=12x+5y=0

Мы видим, что коэффициент при у в первом уравнении и коэффициент при у во втором уравнении – противоположные числа 3 и -3. Cложим почленно первое и второе уравнение и заменим первое уравнение системы тем, которое получится.

2x-5y+x+5y=12+0x+5y=0

3x=12x+5y=0

x=4x+5y=0

x=44+5y=0

x=4y=-45

Ответ: (4;-45).

Решим еще одну систему уравнений

5x+10y=1810x-7y=90

Почленное сложение уравнений ничего нам не даст.

Умножим обе части первого уравнения на -2. Получим

-10x-20y=-3610x-7y=90

Вот теперь можно применить способ сложения

-27y=5410x-7y=90

y=-210x-7y=90

y=-210x+14=90

y=-2x=735

Ответ: (735;-2).

Практикум по решению систем уравнений | Тренажёр по алгебре (8 класс):

По теме: методические разработки, презентации и конспекты

Урок в 9 классе по теме » Решение систем уравнений 2 степени»

Данный урок уместно использовать при подготовке к ГИА….

Методическая разработка урока алгебры в 7 классе «Различные способы решения систем линейных уравнений» способы решения систем уравнений

Урок алгебры в 7 классе направлен на обобщение и систематизацию различных способов решения систем уравнений: метода сравнения, сложения, подстановки, графического метода, метода Крамера, выбора рацион…

Урок-практикум в 9 классе по теме «Решение систем уравнений второй степени»

Урок-практикум в 9 классе по теме «Решение систем уравнений второй степени»…

Графическое решение неравенств c одной переменной. Графический способ решения систем уравнений.

Алгебра. Повторение. Подготовка к ГИА. 9  класс….

Урок-практикум по алгебре в 9 классе по теме «Решение систем уравнений с двумя переменными»

Цель урока: 1)      Рассмотрение различных способов решения систем уравнений.2)      Продолжение  обучению самостоятельной работе с учебник…

Презентации по теме «Системы двух линейных уравнений», «Метод подстановки для решения систем уравнений», «Метод сложения для решения систем уравнений» .

Презентации проедполагает использование при проведении онлайн урока по теме «Системы двух линейных уравнений», «Метод подстановки для решения систем уравнений», «Метод сложени…

Технологическая карта урока алгебры в 9 классе по теме: «Решение систем уравнений второй степени с двумя переменными. Графический способ решения систем уравнений»

1. Разработка технологической карты урока алгебры в 9 классе по теме: «Решение систем уравнений второй степени с двумя переменными. Графический способ решения систем уравнений.2. Технологическая …

Способы решения систем линейных уравнений (Реферат)

Способы решения систем линейных уравнений – очень интересная и важная тема. Системы уравнений и методы их решения рассматриваются в школьном курсе математики, но недостаточно широко. А для того, чтобы перейти к исследованию данной темы, также нужно было познакомиться с темой матриц и определителей. Этот же материал вообще в школьной программе не изучается. Поэтому первая глава моего реферата посвящена теме матриц и определителей. В ней я рассматривала различные действия над матрицами, свойства определителей, метод Гаусса вычисления ранга матрицы, а так же некоторые другие теоретические вопросы. Во второй главе непосредственно рассматриваются системы линейных уравнений и некоторые методы их решения: правило Крамера, метод Гаусса, а так же теорема Кронекера – Капелли. И в той и в другой главах приведены примеры, которые составляют практическую часть моего реферата.

Цель моей работы заключается в том, чтобы изучить различные способы решения систем линейных уравнений для применения их на практике. Для достижения любой цели необходимо выполнить какие-то определенные задачи. Мне нужно выполнить следующие задачи: исследовать литературу по темам матриц, определителей и систем линейных уравнений; изучить современное состояние данного вопроса; отобрать и классифицировать исследуемый материал; а также провести практическую часть работы. Давайте рассмотрим некоторые примеры важнейших моментов этой работы.

Пусть дана система n линейных уравнений с n неизвестными:

a11x1 + a12x2 + …+ a1n xn = b1 ;

a21x1 + a22x2 + …+ a2n xn = b2 ;

……………………………………

an1x1 + an2x2 + …+ annxn = bn ;

a). Если , то система (1) имеет единственное решение,

которое может быть найдено по формулам Крамера: x1=, где

определитель n-го порядка i ( i=1,2,…,n) получается из определителя системы путем замены i-го столбца свободными членами b1 , b2 ,…, bn.

б). Если , то система (1) либо имеет бесконечное множество решений , либо несовместна ,т.е. решений нет. Например:

решить систему уравнений

.

Вычислим определитель системы:

Так как определитель не равен нулю, система уравнений может быть решена по формулам Крамера. Найдем определители ∆x , ∆y:

.

Практическое значение правила Крамера для решения системы n линейных уравнений с п неизвестными невелико, так как при его применении приходится вычислять п +1 определителей n-го порядка: , x1, x2, …,xn. Более удобным является так называемый метод Гаусса. Он применим и в более общем случае системы линейных уравнений, т. е. когда число уравнений не совпадает с числом неизвестных.

Итак, пусть дана система, содержащая m линейных уравнений с п неизвестными:

а11х1 + а12х2 + …+ а1nхn = b1;

а21х1 + а22х2 + …+ а2nхn = b2;

. ……………………………………

аm1х1 + аm2х2 + …+ аmnхn = bm

Метод Гаусса решения системы (19) заключается в последовательном исключении переменных. Например:

Решить методом Гаусса систему уравнений

x1 – 2x2 + x3 + x4 = –1;

3x1 + 2x2 – 3x3 – 4x4 = 2;

2x1x2 + 2x3 – 3x4 = 9;

x1 + 3x2 – 3x3x4 = –1.

Р е ш е н и е. Составим матрицу В и преобразуем ее. Для удобства вычислений отделим вертикальной чертой столбец, состоящий из свободных членов:

1 –2 1 1 –1

B = 3 2 –3 –4 2 .

2 –1 2 –3 9

1 3 –3 –1 –1

Умножим первую строку матрицы В последовательно на 3, 2 и 1 и вычтем соответственно из второй, третьей и четвертой строк. Получим матрицу, эквивалентную исходной:

1 –2 1 1 –1

0 8 –6 –7 5

0 3 0 –5 11

0 5 –4 –2 0

Третью строку матрицы умножим на 3 и вычтем ее из второй строки. Затем новую вторую строку умножим на 3 и на 5 и вычтем из третьей и четвертой строк. Получим матрицу, эквивалентную исходной:

1 –2 1 1 –1

0 –1 –6 8 –28

0 0 –1 0 –3

0 0 0 19 –19

Из коэффициентов последней матрицы составим систему, равносильную исходной:

x1 – 2x2 + x3 + x4 = –1;

x3 = –3;

19x4 = –19.

Решим полученную систему методом подстановки, двигаясь последовательно от последнего уравнения к первому. Из четвертого уравнения x4 = –1, из третьего х3 = 3. Подставив значения х3 и x4 во второе уравнение, найдем x2 = 2. Подставив значения x2, x3, x4 в первое уравнение, найдем x1 = 1.

Теорема совместности Кронекера – Капелли звучит следующим образом: Для того, чтобы система неоднородных линейных уравнений была совместной, необходимо и достаточно, чтобы ранг расширенной матрицы системы был равен рангу её основной матрицы. Рассмотрим следующий пример:

Рассмотрим систему

5x1x2 + 2x3 + x4 = 7;

2x1 + x2 – 4x3 – 2x4 = 1;

x1 – 3x2 + 6x3 – 5x4 = 0.

Ранг основной матрицы этой системы равен 2, так как сцществует отличный от нуля минор второго порядка этой матрицы, например

5 –1 = 7,

2 1

а все миноры третьего порядка равны нулю.

Ранг расширенной матрицы этой системы равен 3, так как существует отличный от нуля минор третьего порядка этой матрицы, например

5 –1 7

2 1 1 = –35.

1 –3 0

Согласно критерию Кронекера – Капелли система несовместна, т.е. не имеет решений.

В процессе работы я узнала много нового: какие действия можно выполнять над матрицами, какой путь решения систем линейных уравнений наиболее простой и быстрый, а так же многие другие теоретические вопросы и провела практические исследования, приводя примеры в тексте.

Тема решения систем линейных уравнений предлагается на вступительных экзаменах в различные математические вузы, на выпускных экзаменах, поэтому умение их решать очень важно.

Реферат может использоваться как учащимися, так и преподавателями в процессе факультативных занятий, как пособие для самостоятельного изучения по теме „Способы решения систем линейных уравнений ”, а также в качестве дополнительного материала.

МОУ Гимназия № 11

Способы решения систем линейных уравнений

Анжеро-Судженск

2004г.

МОУ Гимназия № 11

Способы решения систем линейных уравнений

Реферат по математике

Выполнила:

Ученица 92 класса

Бойко Юлия

Научный

Руководитель:

Клокова Татьяна

Васильевна.

Анжеро-Судженск

2004г.

Содержание:

Презентация — Методы решения систем уравнений

Слайд №2
Цель урока
Продолжить формирование навыков сознательного выбора способа решения системы;
Развивать потребность в нахождении рациональных способов решения;
Воспитывать умение контролировать внимание на всех этапах урока;
Слайд №3
«Математику нельзя изучать, наблюдая как это делает сосед»
Слайд №4
ТЕСТпо теме «Подготовка к ГИА » 7 – 9 классы
Слайд №5
1. Вычислите:
Проверка
Слайд №6
2. Упростите выражение
Проверка
Слайд №7
3. Разложите на множители
Проверка
Слайд №8
4. Выполните умножение
Проверка
Слайд №9
5. Упростите выражение
Проверка
Слайд №10
6. Сколько процентов составляет число 8 от
своего квадрата?
Проверка
Слайд №11
7. Найдите наибольший корень уравнения

Проверка

Слайд №12
8. Решите неравенство
Проверка
Слайд №13
9. График какой функции изображен
на рисунке?
O
x
1
y
Проверка
Слайд №14
Б.
В. 1
Г. При х = — 4/9 выражение не имеет смысла.
А.
10. Найдите значение выражения
Проверка
Слайд №15
A. A = Nt
В.
Г.
Б.
11. Из формулы мощности N=A/t выразите работу A.
Проверка
Слайд №16
А.
В.
Г.
Б.
12. Упростите выражение
Проверка
Слайд №17
13. Какое из данных выражений не равно
А.
Б.
В.
Г.
Проверка
Слайд №18
14. Велосипедист от озера до деревни ехал со скоростью 15км/ч, а обратно – со скоростью 10 км/ч. Сколько времени ушло у него на дорогу от озера до деревни, если на весь путь туда и обратно велосипедист затратил 1 ч?
Пусть х ч – время, затраченное на дорогу от озера до деревни. Какое из уравнений соответствует условию задачи?
А.
Б. 15х = 10(1 – х)
В. 15х + 10(1 – х) = 1
Г. 15(1 – х) = 10х
Проверка
Слайд №19
Ответ: 2.
РЕШЕНИЕ:
Слайд №20
Ответ: 2.
2. Упростите выражение
РЕШЕНИЕ:
Слайд №21
Ответ: 2.
3. Разложите на множители
РЕШЕНИЕ:
Слайд №22
Ответ: 1.
4. Выполните умножение
РЕШЕНИЕ:
Слайд №23
РЕШЕНИЕ:
Ответ: 4.
Слайд №24
6. Сколько процентов составляет число 8 от
своего квадрата?
Решение:
%
Ответ: 3.
Слайд №25
7. Найдите наибольший корень уравнения
РЕШЕНИЕ:
a + b + c = 2 + 3 – 5 = 0, значит х1 = 1, х2 = — 5 : 2 = — 2,5.
Наибольший корень уравнения х = 1.
Ответ: 1.
Слайд №26
8. Решите неравенство
РЕШЕНИЕ:
х
— 0,5
0,5
+
+

Ответ: 2.
Слайд №27
9. График какой функции изображен
на рисунке?
РЕШЕНИЕ:
Ответ: 3.
Слайд №28
10. Найдите значение выражения
РЕШЕНИЕ:
Ответ: Б.
Слайд №29
11. Из формулы мощности N=A/t выразите работу A.
РЕШЕНИЕ:
Ответ: А.
Слайд №30
13. Упростите выражение
РЕШЕНИЕ:
Ответ: А.
Слайд №31
13. Какое из данных выражений не равно
РЕШЕНИЕ:
Ответ: Б.
Слайд №32
14. Велосипедист от озера до деревни ехал со скоростью 15км/ч, а обратно – со скоростью 10 км/ч. Сколько времени ушло у него на дорогу от озера до деревни, если на весь путь туда и обратно велосипедист затратил 1 ч?
Пусть х ч – время, затраченное на дорогу от озера до деревни. Какое из уравнений соответствует условию задачи?
РЕШЕНИЕ:
Если х ч – время затраченное на дорогу от озера до деревни и
все время 1 ч, то на путь обратно велосипедист затратил
(1 – х) ч.
15х км – расстояние от озера до деревни,
10(1 – х) км – расстояние от деревни до озера.
Зная, что расстояния одинаковые, составляем уравнение:
Ответ: Б.
Слайд №33
Алгебра стоит на четырёх китах
Число
Тождество
Функция
Алгебра щедра. Зачастую она дает больше, чем у нее спрашивают
Ж.Даламбер
число
уравнение
тождество
функция
Слайд №34
Определение
Уравнение – это равенство, содержащее одну или несколько переменных
Линейное уравнение с
одной переменной

Линейное уравнение с
двумя переменными

Свойства уравнений
если в уравнении перенести слагаемое из одной части в другую, изменив его знак, то получится уравнение, равносильное данному
если обе части уравнения умножить или разделить на одно и то же отличное от нуля число, то получится уравнение, равносильное данному
Уравнение и его свойства

Слайд №35
Вопросы для размышления
Являются ли данные выражения уравнениями?
a) 5x + 4 = 0; b) 2 + 3x; c) 7x + 3 = 4y; d) 7x + 5y;
Равносильны ли эти уравнения?
2x + 3y = 10 и 2x = 10 – 3y;
10 – 2x = 5y и 10 = 5y – 2x;
Равносильны ли эти уравнения?
2x + y = 3 и 4x +2y = 6;
21x + 15 = 3y и 7x + 5 = 9y;

Выпишите буквы правильных ответов

Слайд №36
Что называется системой уравнений?

Что называется решением системы уравнений?

Что значит – решить систему уравнений?

Слайд №37
Система уравнений и её решение
Определения
Системой уравнений называется некоторое количество уравнений, объединенных фигурной скобкой. Фигурная скобка означает, что все уравнения должны выполняться одновременно
Каждая пара значений переменных, которая одновременно является решением всех уравнений системы, называется решением системы
Решением системы уравнений с двумя переменными называется пара значений переменных, обращающая каждое уравнение системы в верное равенство
Решить систему уравнений — это значит найти все её решения или установить, что их нет
Слайд №38
Сколько решений имеет система уравнений ?
Если
к1 к2 Графики
пересекаются Система имеет единственное решение
Если к1=к2,
b1 b2 Графики параллельны Система не имеет решений
Если к1=к2,
b1=b2 Графики совпадают Система имеет бесконечно много решений
Слайд №39
Является ли пара чисел решением системы
(3;1)
(2;2)
верно
неверно
верно
верно
(3;1) не является решением
(2;2) является решением
Слайд №40
Методы решения систем уравнений
способ подстановки
способ сложения
графический способ
Слайд №41
Решить систему уравнений способом подстановки
y – 2x = 4 ,
7x – y = 1 .
Слайд №42
Решение системы способом подстановки
7х — 2х — 4 = 1;
5х = 5;
х=1;
Ответ: х=1; у=6.
Слайд №43
Способ подстановки (алгоритм)
Из какого-либо уравнения выразить одну переменную через другую
Подставить полученное выражение для переменной в другое уравнение и решить его
Сделать подстановку найденного значения переменной и вычислить значение второй переменной
Записать ответ: х=…; у=… .
Слайд №44
Решить систему уравнений способом сложения
7x + 2y = 1 ,
17x + 6y = -9 .
Слайд №45
Решение системы способом сложения
||·(-3)
+
____________
Ответ: (3; — 10)
Слайд №46
Способ сложения (алгоритм)
Уравнять модули коэффициентов при какой-нибудь переменной
Сложить почленно уравнения системы
Составить новую систему: одно уравнение новое, другое — одно из старых
Решить новое уравнение и найти значение одной переменной
Подставить значение найденной переменной в старое уравнение и найти значение другой переменной
Записать ответ: х=…; у=… .
Слайд №47
Решение системы графическим способом
y=10 — x
y=x+2
Выразим у
через х
Построим график
первого уравнения
у=х+2
Построим график
второго уравнения
у=10 — х
Ответ: (4; 6)
Слайд №48
Графический способ (алгоритм)
Выразить у через х в каждом уравнении
Построить в одной системе координат график каждого уравнения
Определить координаты точки пересечения
Записать ответ: х=…; у=… , или (х; у)
Слайд №49
Задание 1
Строим в одной системе координат графики уравнений системы
А теперь самостоятельно определите решения системы.
Слайд №50
Задание 2
Строим в одной системе координат графики уравнений системы
А теперь самостоятельно определите решения системы.
Слайд №51
Задание 3
3х+2у=18
Перед Вами графики двух уравнений. Запишите систему, определяемую этими уравнениями, и ее решение.
Слайд №52
Перед Вами графики двух уравнений. Запишите систему, определяемую этими уравнениями, и ее решение.
Задание 4
Слайд №53
Применение систем уравнений
1. В настоящее время в компьютерной технологии широко используются электронные таблицы для решения задач управления в промышленности, бизнесе, финансовой деятельности.
Электронная таблица легко позволяет реализовать один из методов вычислительной математики — метод итераций.
Наибольшее применение итерационный метод нашел при решении систем линейных уравнений. К таким системам сводятся задачи анализа электрических цепей, расчета энергий колебательных уровней двухатомных молекул и др. Метод используется и для решения систем нелинейных уравнений: система «хищник-жертва» и др.
2. Решение задачи о месте и времени встречи промыслового рыболовецкого судна с перегрузчиком сводится по сути к решению систем линейных уравнений, использующих данные о координатах судов, их скоростях и метеоусловиях.
Слайд №54
Самостоятельная работа
Решите системы уравнений
Слайд №55
Домашнее задание:

п.6 (определение, примеры)
№ 6.7(а,б)
№ 6.8(а,б)сборник ГИА №3.5

Слайд №56
Спасибо всем за работу
Слайд №57
ИСПОЛЬЗОВАННЫЕ РЕСУРСЫ:

ШАБЛОН – СЕТЬ ТВОРЧЕСКИХ УЧИТЕЛЕЙ «СОЗДАНИЕ ИНТЕРАКТИВНЫХ ТЕСТОВ НА УРОКАХ МАТЕМАТИКИ» Савченко Е. М.
КАРТИНКИ – КОЛЛЕКЦИЯ ОБРАЗОВАТЕЛЬНЫХ РЕСУРСОВ И СЕТЬ ТВОРЧЕСКИХ УЧИТЕЛЕЙ
«АЛГЕБРА» Сборник заданий для подготовки к итоговой аттестации в 9 классе. Москва. Просвещение. 2011.

Решение систем уравнений (одновременных уравнений)

Если у вас есть два разных уравнения с одинаковыми двумя неизвестными в каждом, вы можете решить для обоих неизвестных. Существует три распространенных метода решения: сложение / вычитание, подстановка и построение графика.

Метод сложения / вычитания

Этот метод также известен как метод исключения.

Чтобы использовать метод сложения / вычитания, выполните следующие действия:

  1. Умножьте одно или оба уравнения на некоторое число (а), чтобы число перед одной из букв (неизвестных) в каждом уравнении было одинаковым или прямо противоположным.

  2. Сложите или вычтите два уравнения, чтобы исключить одну букву.

  3. Решите оставшееся неизвестное.

  4. Решите для другого неизвестного, вставив значение неизвестного, найденного в одно из исходных уравнений.

Пример 1

Решите для x и y .

При добавлении уравнений исключаются термины и .

Теперь добавление 5 для x в первое уравнение дает следующее:

Ответ: x = 5, y = 2

Заменяя каждое x на 5 и каждое y на 2 в исходных уравнениях, вы можете увидеть, что каждое уравнение станет истинным.

В примере и пример , существовал уникальный ответ для x и y , который делал каждое предложение одновременно истинным. В некоторых ситуациях вы не получаете однозначных ответов или вообще не получаете ответов. Вы должны знать об этом, когда используете метод сложения / вычитания.

Пример 2

Решите для x и y.

Сначала умножьте нижнее уравнение на 3. Теперь перед y стоит 3 в каждом уравнении.

Уравнения можно вычесть, исключив члены y .

Вставьте x = 5 в одно из исходных уравнений, чтобы найти y .

Ответ: x = 5, y = 3

Конечно, если число перед буквой уже одно и то же в каждом уравнении, вам не нужно изменять ни одно уравнение. Просто сложите или вычтите.

Чтобы проверить решение, замените каждое x в каждом уравнении на 5 и замените каждое y в каждом уравнении на 3.

Пример 3

Решите относительно a и b .

Умножьте верхнее уравнение на 2. Обратите внимание на то, что происходит.

Теперь, если вы вычтете одно уравнение из другого, результат будет 0 = 0.

Это утверждение всегда верно .

Когда это происходит, система уравнений не имеет единственного решения. Фактически, любая замена a и b , которая делает одно из уравнений истинным, также делает истинным другое уравнение.Например, если a = –6 и b = 5, то оба уравнения выполняются.

[3 (- 6) + 4 (5) = 2 И 6 (- 6) + 8 (5) = 4]

В действительности мы имеем только одно уравнение, записанное двумя разными способами. В этом случае второе уравнение на самом деле является первым уравнением, умноженным на 2. Решением этой ситуации является либо исходное уравнение, либо упрощенная форма любого уравнения.

Пример 4

Решите для x и y .

Умножьте верхнее уравнение на 2. Обратите внимание на то, что происходит.

Теперь, если вы вычтете нижнее уравнение из верхнего уравнения, результат будет 0 = 1. Это утверждение никогда не соответствует действительности . Когда это происходит, система уравнений не имеет решения.

В примерах 1–4 только одно уравнение было умножено на число, чтобы числа перед буквой были одинаковыми или противоположными. Иногда каждое уравнение необходимо умножить на разные числа, чтобы числа перед буквой были одинаковыми или противоположными.

Решите для x и y .

Обратите внимание, что не существует простого числа, на которое можно умножить любое уравнение, чтобы получить числа перед x или y , чтобы они стали одинаковыми или противоположными. В этом случае сделайте следующее:

  1. Выберите букву, которую нужно удалить.

  2. Используйте две цифры слева от этой буквы. Найдите наименьшее общее кратное этого значения как желаемое число перед каждой буквой.

  3. Определите, на какое значение необходимо умножить каждое уравнение, чтобы получить это значение, и умножьте уравнение на это число.

Предположим, вы хотите исключить x . Наименьшее общее кратное 3 и 5, число перед x , равно 15. Первое уравнение нужно умножить на 5, чтобы получить 15 перед x . Второе уравнение нужно умножить на 3, чтобы получить 15 перед x .

Теперь вычтите второе уравнение из первого, чтобы получить следующее:

На этом этапе вы можете либо заменить y на и решить для x (метод 1, который следует ниже), либо начать с двух исходных уравнений и исключить y , чтобы решить для x (метод 2, который следует).

Метод 1

Используя верхнее уравнение: замените y на и решите относительно x .

Метод 2

Исключаем y и решаем относительно x .

Наименьшее общее кратное 4 и 6 равно 12. Умножьте верхнее уравнение на 3, а нижнее уравнение на 2.

Теперь сложите два уравнения, чтобы исключить y .

Решение: x = 1 и.

Метод замещения

Иногда система легче решается методом подстановки . Этот метод включает замену одного уравнения в другое.

Пример 6

Решите для x и y.

Из первого уравнения замените ( y + 8) на x во втором уравнении.

( y + 8) + 3 y = 48

Теперь решите г. Упростите, объединив и .

Теперь вставьте значение y , 10, в одно из исходных уравнений.

Ответ: y = 10, x = 18

Проверьте решение.

Пример 7

Решите для x и y , используя метод подстановки.

Сначала найдите уравнение, в котором перед буквой стоит цифра «1» или «- 1». Решите эту букву с точки зрения другой буквы.

Затем действуйте как в примере 6.

В этом примере в нижнем уравнении перед числами y стоит «1».

Решите относительно y в терминах x .

Замените 4 x -17 вместо y в верхнем уравнении, а затем решите относительно x .

Замените x на 4 в уравнении y — 4 x = –17 и решите относительно y .

Решение: x = 4, y = –1.

Проверьте решение:

Графический метод

Другой метод решения уравнений — это графическое отображение каждого уравнения на координатном графике. Координаты перекрестка будут решением системы. Если вы не знакомы с построением координатных графиков, внимательно просмотрите статьи по координатной геометрии, прежде чем пытаться использовать этот метод.

Пример 8

Решите систему, построив график.

Сначала найдите три значения для x и y , которые удовлетворяют каждому уравнению. (Хотя для определения прямой необходимы только две точки, поиск третьей точки — хороший способ проверки.) Ниже приведены таблицы значений x и y :

Теперь изобразите две линии на координатной плоскости, как показано на рисунке 1.

Точка пересечения двух линий (4, 0) — это решение системы.

Если линии параллельны, они не пересекаются, и, следовательно, для этой системы нет решения.

Рис. 1. График из линий x = 4 + y и x — 3 y = 4, обозначающих решение.

Пример 9

Решите систему, построив график.

Найдите три значения для x и y , которые удовлетворяют каждому уравнению.

3 x + 4 y = 2 6 x + 8 y = 4

Ниже приведены таблицы значений x и y .См. Рисунок 2.

х

л

0

2

— 1

4

х

л

0

2

— 1

4

Обратите внимание, что одинаковые точки удовлетворяют каждому уравнению.Эти уравнения представляют собой одну и ту же линию.

Следовательно, решение не единственное. Решение — это все точки на линии.

Следовательно, решением является любое уравнение линии, поскольку они оба представляют одну и ту же линию.

Это похоже на пример когда это было сделано с использованием метода сложения / вычитания.

Рисунок 2. График из линий 3 x + 4 y = 2 и 6 x + 8 y = 4, указывающих решение.

Пример 10

Решите систему, построив график.

Найдите три значения для x и y , которые удовлетворяют каждому уравнению. См. Следующие таблицы значений x и y :

х

л

0

1

2

4

-2

х

л

0

2

2

4

–1

Обратите внимание на то, что на рисунке 3 два графика параллельны.Они никогда не встретятся. Следовательно, у этой системы уравнений нет решения.

Для этой системы уравнений не существует решения.

Это похоже на пример выполняется методом сложения / вычитания.

Рисунок 3. График из линий 3 x + 4 y = 4 и 6 x + 8 y = 16, указывающих решение.

Метод исключения для решения линейных систем (Алгебра 1, Системы линейных уравнений и неравенств) — Mathplanet

Другой способ решения линейной системы — использовать метод исключения.В методе исключения вы либо складываете, либо вычитаете уравнения, чтобы получить уравнение с одной переменной.

Когда коэффициенты одной переменной противоположны, вы добавляете уравнения, чтобы исключить переменную, а когда коэффициенты одной переменной равны, вы вычитаете уравнения, чтобы исключить переменную.


Пример

$$ \ begin {matrix} 3y + 2x = 6 \\ 5y-2x = 10 \ end {matrix} $$

Мы можем исключить переменную x, добавив два уравнения.

$$ 3y + 2x = 6 $$

$$ \ underline {+ \: 5y-2x = 10} $$

$$ = 8лет \: \: \: \: \; \; \; \; = 16 $$

$$ \ begin {matrix} \: \: \: y \: \: \: \: \: \; \; \; \; \; = 2 \ end {matrix} $$

Теперь значение y можно подставить в любое из исходных уравнений, чтобы найти значение x

$$ 3y + 2x = 6 $$

$$ 3 \ cdot {\ color {зеленый} 2} + 2x = 6 $$

$$ 6 + 2x = 6 $$

$$ x = 0 $$

Решение линейной системы есть (0, 2).

Чтобы избежать ошибок, перед началом исключения убедитесь, что все одинаковые члены и знаки равенства находятся в одних и тех же столбцах.

Если у вас нет уравнений, в которых вы можете исключить переменную путем сложения или вычитания, вы можете непосредственно начать с умножения одного или обоих уравнений на константу, чтобы получить эквивалентную линейную систему, в которой вы можете исключить одну из переменных путем сложения. или вычитание.

Пример

$$ \ begin {matrix} 3x + y = 9 \\ 5x + 4y = 22 \ end {matrix} $$

Начните с умножения первого уравнения на -4 так, чтобы коэффициенты y были противоположны

$$ \ color {зеленый} {-4 \} \ cdot \ left (3x + y \ right) = 9 \ cdot {\ color {green} {-4} $$

$$ 5x + 4y = 22 $$

$$ — 12x-4y = -36 $$

$$ \ underline {+ 5x + 4y = 22} $$

$$ = — 7x \: \: \: \: \: \: \: \: \: \: = -14 $$

$$ \ begin {matrix} \: \: \; \: \: x \: \: \: \: \: \: \: \: \: \: \: = 2 \ end {matrix} $$

Подставьте x в любое из исходных уравнений, чтобы получить значение y

$$ 3x + y = 9 $$

$$ 3 \ cdot {\ color {зеленый} 2} + y = 9 $$

$$ 6 + y = 9 $$

$$ y = 3 $$

Решение линейной системы: (2, 3)


Видеоурок

Решите линейную систему, используя метод исключения

$$ \ left \ {\ begin {matrix} 2y — 4x = 2 \\ y = -x + 4 \ end {matrix} \ right $$

Как решить систему линейных уравнений

В математике линейное уравнение — это уравнение, которое содержит две переменные и может быть нанесено на график в виде прямой линии.Система линейных уравнений — это группа из двух или более линейных уравнений, содержащих один и тот же набор переменных. Системы линейных уравнений могут использоваться для моделирования реальных проблем. Их можно решить несколькими способами:

  1. Графики
  2. Замена
  3. Исключение добавлением
  4. Исключение вычитанием

Графики

Фотография Эрика Раптоша / Смешанные изображения / Getty Images

Построение графиков — один из простейших способов решения системы линейных уравнений.Все, что вам нужно сделать, это изобразить каждое уравнение в виде линии и найти точки пересечения линий.

Например, рассмотрим следующую систему линейных уравнений, содержащую переменные x и y :


y = x + 3
y = -1 x — 3

Эти уравнения уже записаны в форме пересечения наклона, что упрощает их графическое отображение. Если бы уравнения не были записаны в форме пересечения наклона, вам сначала нужно было бы упростить их.Как только это будет сделано, решение для x и y потребует всего нескольких простых шагов:

1. Изобразите оба уравнения.

2. Найдите точку пересечения уравнений. В этом случае ответ (-3, 0).

3. Убедитесь, что ваш ответ правильный, подставив значения x = -3 и y = 0 в исходные уравнения.


y = x + 3
(0) = (-3) + 3
0 = 0

y = -1 x — 3
0 = -1 (-3) — 3
0 = 3–3
0 = 0

Замена

Другой способ решить систему уравнений — это подстановка.С помощью этого метода вы существенно упрощаете одно уравнение и включаете его в другое, что позволяет исключить одну из неизвестных переменных.

Рассмотрим следующую систему линейных уравнений:


3 x + y = 6
x = 18-3 y

Во втором уравнении уже выделено x . Если бы это было не так, нам сначала нужно было бы упростить уравнение, чтобы выделить x .Выделив x во втором уравнении, мы можем затем заменить x в первом уравнении эквивалентным значением из второго уравнения: (18 — 3y) .

1. Замените x в первом уравнении заданным значением x во втором уравнении.


3 ( 18 — 3 года ) + лет = 6

2. Упростите каждую часть уравнения.


54 — 9 y + y = 6
54 — 8 y = 6

3.Решите уравнение относительно y .

54-8 y -54 = 6-54
-8 y = -48
-8 y / -8 = -48 / -8
у = 6

4. Подставьте y = 6 и решите относительно x .


x = 18-3 y
x = 18-3 (6)
x = 18–18
х = 0

5. Убедитесь, что (0,6) является решением.


x = 18-3 y
0 = 18 — 3 (6)
0 = 18-18
0 = 0

Исключение добавлением

Если приведенные вам линейные уравнения записаны с переменными с одной стороны и константой с другой, самый простой способ решить систему — исключить.

Рассмотрим следующую систему линейных уравнений:


x + y = 180
3 x + 2 y = 414

1. Сначала напишите уравнения рядом друг с другом, чтобы можно было легко сравнить коэффициенты с каждой переменной.

2. Затем умножьте первое уравнение на -3.


-3 (х + у = 180)

3. Почему мы умножили на -3? Добавьте первое уравнение ко второму, чтобы узнать.


-3x + -3y = -540
+ 3х + 2у = 414
0 + -1y = -126

Теперь мы удалили переменную x .

4. Найдите переменную y :


y = 126

5. Подставьте y = 126, чтобы найти x .


x + y = 180
х + 126 = 180
х = 54

6. Убедитесь, что (54, 126) правильный ответ.


3 x + 2 y = 414
3 (54) + 2 (126) = 414
414 = 414

Исключение вычитанием

Другой способ решения методом исключения — это вычесть, а не сложить данные линейные уравнения.

Рассмотрим следующую систему линейных уравнений:


y — 12 x = 3
y -5 x = -4

1. Вместо того, чтобы складывать уравнения, мы можем вычесть их, чтобы исключить y .


y — 12 x = 3
— ( y -5 x = -4)
0–7 x = 7

2. Решите относительно x .


-7 х = 7
х = -1

3. Подставьте x = -1, чтобы найти y .


y — 12 x = 3
y — 12 (-1) = 3
y + 12 = 3
y = -9

4.Убедитесь, что (-1, -9) — правильное решение.


(-9) — 5 (-1) = -4
-9 + 5 = -4
-4 = -4

Систем уравнений в SAT Math: Подготовка к алгебре и практика

Конечно, вы хорошо разбирались в уравнениях с одной переменной, и теперь они не проблема, но что вы делаете, когда вам предлагают сразу несколько уравнений и несколько переменных? Это то, что мы называем «системами уравнений», и, к счастью для нас, это чрезвычайно предсказуемые типы проблем с множеством методов их решения.В зависимости от того, как вам больше всего нравится работать, вы можете выбрать свое собственное приключение, когда дело доходит до задач системы уравнений.

Но прежде чем вы выберете метод, который лучше всего подходит вам (или конкретной проблеме), давайте рассмотрим все возможные варианты, которые у вас есть, а также типы вопросов, которые вы увидите в день тестирования. Эти вопросы всегда будут появляться один или два раза в любом заданном тесте, поэтому лучше понять все стратегии, которые есть в вашем распоряжении.

Это будет ваше полное руководство по вопросам системы уравнений — что это такое, множество различных способов их решения и как вы увидите их на SAT.

Что такое системы уравнений?

Системы уравнений — это набор из двух (или более) уравнений, которые имеют две (или более) переменные. Уравнения зависят друг от друга и могут быть решены только с информацией, которую предоставляет каждое.

Большую часть времени на тесте SAT вы будете видеть систему уравнений, которая включает два уравнения и две переменные, но, конечно же, нет ничего необычного в том, что вы увидите три уравнения и / или три переменные в любом количестве комбинаций. .

Системы уравнений также можно решать множеством способов. Как всегда в случае с SAT, то, как вы решите свои проблемы, в основном зависит от того, как вы предпочитаете работать, а также от того, сколько времени у вас есть, чтобы посвятить проблеме.

Три метода решения задачи системы уравнений:

# 1 : построение графика
# 2 : подстановка
# 3 : вычитание

Давайте рассмотрим каждый метод и увидим их в действии на примере той же системы уравнений.

Для нашего примера допустим, что наша данная система уравнений:

$$ 2y + 3x = 38 $$

$$ y — 2x = 12 $$

Метод решения 1: построение графиков

Будет только одно решение для системы уравнений, и это одно решение будет пересечением двух линий. Чтобы построить график наших уравнений, мы должны сначала преобразовать каждое уравнение в форму с пересечением наклона. Если вы знакомы с линиями и уклонами, вы знаете, что форма пересечения откоса выглядит так:

$ y = mx + b

$

Итак, давайте представим наши два уравнения в форме пересечения наклона.

$ 2y + 3x = 38 $

$ 2y = -3x + 38 $

$ y = {-3/2} x + 19

$

и

$ y — 2x = 12 $

$ y = 2x + 12 $

Теперь давайте изобразим каждое уравнение на графике, чтобы найти точки пересечения.

После того, как мы изобразили наше уравнение, мы увидим, что пересечение находится в (2, 16).

Итак, наши окончательные результаты: $ x = 2 $ и $ y = 16 $

Метод решения 2: Замена

Чтобы решить нашу систему уравнений с помощью подстановки, мы должны выделить одну переменную в одном из уравнений, а затем использовать эту найденную переменную для второго уравнения , чтобы найти оставшуюся переменную.

Например, у нас есть два уравнения,

$ 2y + 3x = 38 $

$ y — 2x = 12 $

Итак, давайте выберем только из уравнений, а затем выделим одну из переменных.

В этом случае давайте выберем второе уравнение и выделим наше значение $ y $.

$ y — 2x = 12 $

$ y = 2x + 12 $

Затем мы должны подставить найденную переменную во второе уравнение. (В этом случае, поскольку мы использовали второе уравнение для выделения нашего $ y $, нам нужно вставить это значение $ y $ в первое уравнение .)

$ 2y + 3x = 38 $

$ 2 (2x + 12) + 3x = 38 $

$ 4x + 24 + 3x = 38 $

$ 24 + 7x = 38 $

$ 7x = 14 $

долл. США x = 2 долл. США

долл. США

И, наконец, вы можете найти числовое значение для вашей первой переменной ($ y $), подставив числовое значение для вашей второй переменной ($ x $) в любое уравнение.

$ 2y + 3x = 38 $

$ 2y + 3 (2) = 38 $

$ 2y + 6 = 38 $

$ 2y = 32 $

$ у = 16 $

или

$ y — 2x = 12 $

$ г — 2 (2) = 12 $

$ г — 4 = 12 $

$ у = 16 $

В любом случае, вы нашли значение ваших $ x $ и $ y $.

Опять же, $ x = 2 $ и $ y = 16 $

Метод решения 3: вычитание

В качестве последнего метода решения систем уравнений вы можете полностью вычесть одну из переменных, чтобы найти значение второй переменной. Мы делаем это, вычитая одно уравнение целиком из другого полного уравнения.

Обратите внимание, что вы можете сделать это только в том случае, если рассматриваемые переменные (та, которую вы хотите исключить) точно такие же .Если они НЕ совпадают, тогда мы должны сначала умножить все уравнение на необходимую сумму, чтобы из получилось из них одинаковыми.

В случае наших двух уравнений ни одна из наших переменных не равна.

$ 2y + 3x = 38 $

$ y — 2x = 12 $

В этом случае давайте решим вычесть наши значения $ y $ и отменить их. Это означает, что мы должны сначала сделать их равными , умножив наше второе уравнение на 2, чтобы оба значения $ y $ совпали.

$ 2y + 3x = 38 $

$ y — 2x = 12 $

Стало:

$ 2y + 3x = 38 $ (Это первое уравнение остается неизменным)

и

$ 2 (y — 2x = 12) $ => $ 2y — 4x = 24 $ (Все уравнение умножается на 2)

А теперь мы можем сократить наши значения $ y $, вычтя все второе уравнение из первого.

$ 2y + 3x = 38 $

$ 2y — 4x = 24 $

———————

$ 3x — -4x = 14 $

$ 7x = 14 $

долл. США x = 2 долл. США

долл. США

Теперь, когда мы изолировали наше значение $ x $, мы можем вставить его в любое из наших двух уравнений, чтобы найти значение $ y $.

$ 2y + 3x = 38 $

$ 2y + 3 (2) = 38 $

$ 2y + 6 = 38 $

$ 2y = 32 $

$ у = 16 $

или

$ y — 2x = 12 $

$ г — 2 (2) = 12 $

$ г — 4 = 12 $

$ у = 16 $

Наши окончательные результаты снова равны $ x = 2 $ и $ y = 16 $.

Хотя есть много способов решить ваши проблемы, не позволяйте этим знаниям ошеломить вас; с практикой вы найдете лучший метод решения для вас.

Независимо от того, какой метод мы используем для решения наших проблем, система уравнений будет иметь либо одно решение, — это означает, что каждая переменная будет иметь числовое значение, — без решения, либо бесконечные решения .

Чтобы система уравнений имела бесконечных решений , каждая система фактически идентична. Это означает, что это и та же линия .

Чтобы система уравнений имела нет решение , значения $ x $ будут равны, если для значений $ y $ установлено значение 1 (что означает, что обе переменные — $ x $ и $ y $ — будет равно).Причина, по которой это правда, заключается в том, что в результате получатся две параллельные линии, так как линии будут иметь одинаковый наклон. У системы нет решения, потому что две линии никогда не встретятся и, следовательно, не имеют точки пересечения.

Например,

Поскольку наша система не будет иметь решения, когда и наши значения $ y $, и наши значения $ x $ равны , равным , это означает, что не будет решения, в котором мы исключили бы обе наши переменные путем их отмены.

В этом случае наиболее целесообразным решением данной проблемы будет вычитание.Почему? Мы можем видеть это, потому что два значения $ x $ ($ 2x $ и $ 4x $) кратны друг другу, поэтому мы можем легко перемножить одно уравнение, чтобы уравнять их.

$ 2x — 5лет = 8 $

$ 4x + ky = 17 $

Теперь давайте умножим верхнее уравнение, чтобы сравнять наши значения $ x $. Итак, системная пара,

$ 2 (2x — 5лет = 8) $

$ 4x + ky = 17 $

Становится,

$ 4x — 10лет = 16) $

$ 4x + ky = 17 $

———————-

$ -10лет — ky = -1 $

Чтобы НЕТ решения, наши два значения $ y $ должны уравновеситься до нуля.Итак, давайте установим наши два значения $ y $ равными друг другу:

$ -10y — ky = 0 $

$ -ки = 10лет $

$ k = -10 $

Наше значение $ k $ должно быть равным -10, чтобы наша система уравнений не имела решения.

Наш окончательный ответ — A , -10.

[Примечание: не попадайтесь на удочку ответа +10! Вы все еще вычитаете свою систему уравнений, поэтому внимательно следите за своими отрицаниями.]

Кроме того, не беспокойтесь об этом, если вас расстраивает или сбивает с толку попытка решить, какой из трех методов решения «лучше всего» подходит для конкретной проблемы! Вы почти всегда сможете решить свои задачи по системе уравнений, независимо от того, какой метод вы выберете.

Например, вы могли бы также выбрать график этого вопроса. Если бы вы это сделали, вам сначала нужно было бы преобразовать каждое уравнение в форму пересечения наклона:

$ 2x — 5лет = 8 $

$ 4x + ky = 17 $

$ 2x — 5лет = 8 $

$ -5y = -2x + 8 $

$ y = 2/5 (x) + 8 $

и

$ 4x + ky = 17 $

$ ky = -4x + 17 $

$ y = {-4 / k} (x) + 17 $

Теперь мы знаем, что система уравнений не будет иметь решения только тогда, когда каждая переменная уравновесится до нуля, поэтому давайте приравняем наши две переменные $ x $, чтобы найти $ k $.

$ 2/5 (x) = {-4 / k} (x)

$

$ 2/5 = {-4} / тыс. $

{2k} долл. США / 5 = -4

долл. США

2 тыс. Долл. = -20

долл. США

$ k = -10 $

Опять же, наше значение $ k $ равно -10.

Наш окончательный ответ — A , -10.

Как видите, никогда не бывает «лучшего» метода решения вопроса с системой уравнений, только метод решения, который вам больше всего нравится.


Все дороги ведут в Рим, поэтому не беспокойтесь, пытаясь найти «правильный» метод решения проблем вашей системы.

Типичные вопросы по системам уравнений

Большинство вопросов о системе уравнений в SAT сообщают вам, что это система уравнений, за счет явного использования слов «системы уравнений» в самом вопросе.

(мы рассмотрим, как решить этот вопрос позже в руководстве)

Другие задачи просто представят вам несколько уравнений с общими переменными и попросят вас найти значение одной из переменных или даже комбинацию переменных (например, значение $ x + y $ или $ x — y $).

(мы рассмотрим, как решить этот вопрос позже в руководстве)

И, наконец, последний тип вопроса о системе уравнений попросит вас найти числовое значение переменной, для которой НЕТ решения, как в предыдущем примере.

Готовы выйти за рамки простого чтения о SAT? Тогда вам понравится бесплатная пятидневная пробная версия для нашей программы SAT Complete Prep . Разработанная и написанная экспертами PrepScholar SAT , наша программа SAT настраивается в соответствии с вашим уровнем навыков по более чем 40 вспомогательным навыкам, чтобы вы могли сосредоточить свое обучение на том, что принесет вам наибольший набор баллов.

Нажмите кнопку ниже, чтобы попробовать!

Стратегии решения вопросов по системам уравнений

Все вопросы по системам уравнений могут быть решены с помощью тех же методов, которые мы описали выше, но есть дополнительные стратегии, которые вы можете использовать для наиболее точного и целесообразного решения ваших вопросов.

# 1: Для начала найдите переменную, которая уже является наиболее изолированной

Конечная цель — найти значения всех переменных, но мы можем сделать это, только найдя для начала одну переменную.Самый простой способ решить эту одну переменную — выделить (или исключить) переменную, которая имеет наименьшее количество коэффициентов или, по-видимому, является наиболее изолированной.

Например,

$ 5x — 3 года = -13 $

$ 2x + y = 19 $

Если мы используем подстановку, нам проще всего сначала выделить значение $ y $ во втором уравнении. Это уже самая изолированная переменная, так как у нее нет коэффициентов, и поэтому нам не придется иметь дело с дробями, как только мы заменим ее значение в первом уравнении.

Если, с другой стороны, мы использовали вычитание, все равно лучше нацелить и исключить наши значения $ y $. Почему? Потому что у нас есть $ 3y $ и $ y $, что означает, что нам нужно только умножить второе уравнение на 3, чтобы сопоставить наши значения $ y $. Если бы мы поставили цель и исключили наши значения $ x $, нам пришлось бы умножить и уравнений — первое на 2, а второе на 5, чтобы наши значения $ x $ совпадали.

Хотя вы всегда можете найти свои решения, независимо от того, какие переменные вы решите изолировать или устранить, всегда приятно сэкономить время, энергию и нервы (не говоря уже о том, чтобы избежать возможных ошибок), выбрав в первую очередь простые решения.

# 2: Практикуйте все три метода решения, чтобы определить, какой из них наиболее удобен для вас

Лучший способ решить, какая система решения уравнений подходит вам, , лучше всего — это практиковаться над несколькими задачами (хотя это поможет вашей гибкости, если вы научитесь использовать все доступные методы решения , даже если один или два подходит вам лучше, чем другие).

Когда вы проверяете себя по системным вопросам, постарайтесь решить каждый из них, используя более одного метода, чтобы увидеть, какой из них наиболее удобен для вас лично.

# 3: Используйте вычитание для вопросов, требующих поиска более чем одной переменной

В большинстве вопросов по системам уравнений с множественными переменными вам будет предложено найти $ x + y $ или $ x — y $, которые почти всегда легче всего найти с помощью метода вычитания.

Также наиболее полезно использовать метод вычитания, когда у нас есть трех или более переменных, особенно, когда это комбинация нескольких переменных И трех или более переменных.

Мы увидим этот вид проблемы в действии в следующем разделе.

Готовы взяться за решение системных проблем и проверить свои стратегии?

Проверьте свои знания

Теперь давайте проверим ваши знания системы уравнений на реальных математических вопросах SAT.

Ответы: 300, E, 12

Ответ Пояснения:

1. Как мы описали в нашем разделе стратегии, почти всегда легче всего найти значение нескольких переменных, используя метод вычитания (хотя, опять же, это не способ , а только способ ).

У нас , но несколько ограничен, так как у нас есть три переменные и только два уравнения. Почему это важно? Что ж, мы можем найти индивидуальные значения для каждой переменной, если у нас есть то же количество уравнений, что и у нас есть переменные, но в данном случае это не так. Это означает, что нам нужно использовать решение, которое даст нам $ x + y $, поскольку мы не можем найти значение $ x $ или $ y $ в одиночку.

Итак, воспользуемся вычитанием.

Для этого мы должны вычесть одинаковые переменные, и, к счастью для нас, оба уравнения имеют одно значение $ x + y $.Это означает, что мы можем изолировать нашу переменную $ z $.

$ x + y + 3z = 600 $

$ x + y + z = 400 $

Итак, давайте вычтем их.

$ x + y + 3z = 600 $

$ x + y + z = 400 $

————————-

$ 2z = 200 $

$ z = 100 $

Теперь, когда у нас есть значение $ z $, мы можем заменить его в любом из уравнений, чтобы найти значение $ x + y $.

Поскольку всегда проще всего использовать наиболее изолированную переменную (меньше математики для нас!), Позвольте нам подставить наше второе уравнение для нашего значения $ z $.

$ x + y + z = 400 $

$ x + y + 100 = 400 $

$ x + y = 300 $

Наш окончательный ответ для значения $ x + y $ — 300.

Обратите внимание, однако, что если вы предпочитаете использовать замену, вы определенно можете это сделать.

Поскольку мы пытаемся найти $ x + y $, давайте выделим его как желаемую переменную в одном из наших уравнений.

$ x + y + 3z = 600 $

$ x + y + z = 400 $

Давайте воспользуемся нашим первым уравнением.

$ x + y + 3z = 600 $

$ x + y = 600 — 3z $

А теперь мы можем подставить наше значение $ x + y $ во второе уравнение.

$ x + y + z = 400 $

$ (600 — 3z) + z = 400 $

600 $ — 2z = 400 $

$ -2z = -200 $

$ z = 100 $

Теперь, когда мы нашли наше значение для $ z $, мы можем подставить его в любое уравнение, чтобы найти числовое значение для нашего $ x + y $.

Воспользуемся для этого вторым уравнением.Почему второе? Потому что каждое значение уже является наиболее изолированным и с ним легче всего работать, но каждый вопрос будет работать в любом случае.

$ x + y + z = 400 $

$ x + y + 100 = 400 $

$ x + y = 300 $

И снова наш окончательный ответ: $ x + y = 300 $

Как видите, вам подойдет любой метод — все зависит от того, как вы любите работать.

2. Опять же, хотя не , а только способ решения нашей проблемы, проще всего использовать вычитание, когда у нас есть три или более переменных в наших уравнениях или мы пытаемся найти комбинацию переменных (в данном случае значение $ y + z $).В данном случае у нас есть и то, и другое, поэтому давайте воспользуемся вычитанием.

$ 3x + 2y + 2z = 19 $

$ 3x + y + z = 14 $

Наши значения $ x $ идентичны, поэтому давайте просто вычтем второе уравнение из первого.

$ 3x + 2y + 2z = 19 $

$ 3x + y + z = 14 $

—————————

$ y + z = 5 $

Наш окончательный ответ: E , $ y + z = 5 $

.

3. В этом случае давайте воспользуемся методом подстановки, чтобы выделить одно из наших значений и вставить его в одно из других уравнений нашей системы.

Уравнения, которые нам даны:

$ x = 3v $

$ v = 4t $

$ x =

pt $

$ v $ уже изолирован, поэтому давайте вернем его в наше первое уравнение.

$ v = 4t $

$ x = 3v $

$ x = 3 (4т)

$

$ x = 12т $

Теперь нам также сказали, что $ x = pt $, поэтому мы можем приравнять эти два выражения.

$ x = 12т $

$ x =

pt $

12t =

pt $

Поскольку 12 и $ p $ действуют как коэффициенты (числа перед переменной) для $ t $, мы можем видеть, что они равны.

Это означает, что $ p = 12 $

Наш окончательный ответ — 12.


Вы сделали это! Воздушные шары и конфетти для вас!

Итоги

Как видите, системы уравнений — одни из самых универсальных проблем, когда дело доходит до методов их решения (хотя сами проблемы не так уж и разнообразны). Хотя вы можете решить многие задачи на тесте SAT различными способами, большинство из них не так гибко, поэтому будьте готовы к тому, что у вас есть много вариантов, как поступать с вопросами по системам уравнений.

После того, как вы попрактикуетесь и познакомитесь с этими типами вопросов, вы найдете лучший для себя метод — ваши сильные стороны и время — для прохождения теста. И довольно скоро вы сможете задавать вопросы по системам уравнений разными способами, с завязанными глазами и руками за спиной (хотя, честно говоря, никто не знает, зачем вам это нужно).

Что дальше?

Системы — это совсем несложно, говорите? Вы говорите, что готовы к математическим задачам? Что ж, вам повезло! У нас больше руководств по математике, чем вы можете представить, и все они охватывают важные аспекты математической секции SAT.Линии и углы, многоугольники, целые числа, соотношения … любая тема, которую вам нужно освежить, находится у вас под рукой, поэтому максимально используйте свое время и энергию и нацельтесь на любую из ваших известных проблемных областей до дня теста.

Хотите знать самые полезные стратегии для математических задач SAT? Ознакомьтесь с нашими руководствами по добавлению ответов и добавлению чисел, которые помогут уточнить подавляющее большинство ваших вопросов по SAT математике.

Хотите получить наивысший балл? Не ищите ничего, кроме нашего руководства по получению идеальных 800 баллов по математике SAT, написанного отличником.

Хотите улучшить свой результат SAT на 160 баллов?

Ознакомьтесь с нашей лучшей в своем классе онлайн-программой подготовки к SAT. Мы гарантируем возврат ваших денег , если вы не улучшите свой SAT на 160 или более баллов.

Наша программа полностью интерактивна, и она адаптирует то, что вы изучаете, к вашим сильным и слабым сторонам. Если вам понравилось это руководство по математической стратегии, вам понравится наша программа. Наряду с более подробными уроками вы получите тысячи практических задач, организованных по индивидуальным навыкам, чтобы вы учились наиболее эффективно.Мы также дадим вам пошаговую программу, которой нужно следовать, чтобы вы никогда не запутались, что изучать дальше.

Воспользуйтесь нашей 5-дневной бесплатной пробной версией:

Решение систем линейных уравнений с помощью графического рабочего листа

© e 82×0 M1G26 YKNuCt la X Sdo wf9tRwPaHrse f ULMLGCM.8 R 0A 8l HlD rHiNguh 8t3s 0 Krse 0s qe brtv peZdiqn. h Рабочий лист ООО «Кута Софтвер» Kuta Software — Бесконечная алгебра 1 Имя _____ Решение систем уравнений по дате построения графика _____ Период ____

A2.5.4. Решать системы линейных уравнений и неравенств с двумя переменными путем подстановки, построения графиков и использования матриц с тремя переменными;

Систему линейных уравнений можно решить несколькими различными способами, в том числе с помощью построения графиков, подстановки и исключения. Система линейных уравнений — это группа из двух или более линейных уравнений, содержащих один и тот же набор переменных. Системы линейных уравнений могут использоваться для …

Для решения систем линейных уравнений с помощью построения графиков выполните следующие шаги: Поскольку одновременные уравнения включают в себя два уравнения, нанесите их на график и посмотрите, где на графике они пересекаются друг с другом. .Точка пересечения координат x и y будет решением системы уравнений.

Класс 8 »Введение Распечатайте эту страницу. В 8-м классе учебное время должно быть сосредоточено на трех критических областях: (1) формулирование и рассуждение о выражениях и уравнениях, включая моделирование связи двумерных данных с линейным уравнением и решение линейных уравнений и систем линейных уравнений; (2) понимание концепции функции и использование функций для количественного описания…

Cloze: Завершите абзац, заполнив пропущенные термины. Решение систем уравнений с помощью построения графиков. Определите решение данных графиков. Для отсутствия решения введите «нет решения», а для бесконечных решений введите «бесконечные решения».

Решение систем уравнений путем построения графиков Хотя системы двух линейных уравнений с двумя неизвестными могут быть решены с помощью алгебры, это также возможно для систем уравнений путем построения графиков каждого уравнения в системе.

Имя Решение систем линейных уравнений Решайте с помощью построения графиков.Построение графиков CD-3732 2. 4. 6. 8. 3. 5. x-7 (25) (a 3 6 12 -2x + y O © 1995 Kelley Wingate Publications, Inc.

Имя Решение систем линейных уравнений Решить с помощью графиков. Построение графиков CD-3732 2. 4. 6. 8. 3. 5. x-7 (25) (a 3 6 12 -2x + y O © 1995 Kelley Wingate Publications, Inc.

2.4 Решение Системы уравнений с двумя переменными — промежуточная алгебра

Цели обучения

  • Определение и классификация решений систем линейных уравнений
    • Распознавать непротиворечивые и непоследовательные, зависимые и независимые системы линейных уравнений
    • Определить, является ли упорядоченная пара решением системы линейных уравнений
  • Методы решения систем
    • Использовать замену для алгебраического решения системы
    • Распознавать несовместимость системы с алгебраическими результатами
    • Найдите точку безубыточности для системы затрат и доходов
    • Напишите систему уравнений на основе данных о посещаемости и расходах
    • Решите систему методом исключения
    • Решите систему методом исключения, когда требуется умножение
    • Распознавать, когда система зависит, используя алгебраические результаты

Система линейных уравнений состоит из двух или более линейных уравнений, состоящих из двух или более переменных, так что все уравнения в системе рассматриваются одновременно.Чтобы найти единственное решение системы линейных уравнений, мы должны найти числовое значение для каждой переменной в системе, которое будет удовлетворять всем уравнениям в системе одновременно. Некоторые линейные системы могут не иметь решения, а другие могут иметь бесконечное количество решений. Чтобы линейная система имела единственное решение, должно быть по крайней мере столько же уравнений, сколько переменных. Даже в этом случае это не гарантирует уникального решения.

В этом разделе мы рассмотрим системы линейных уравнений с двумя переменными, которые состоят из двух уравнений, содержащих две разные переменные.Например, рассмотрим следующую систему линейных уравнений с двумя переменными.

\ (\ begin {align *} 2x + y & = 15 \\ 3x-y & = 5 \ end {align *} \)

Решение системы линейных уравнений с двумя переменными — это любая упорядоченная пара, которая удовлетворяет каждому уравнению независимо. В этом примере упорядоченная пара (4, 7) является решением системы линейных уравнений. Мы можем проверить решение, подставив значения в каждое уравнение, чтобы увидеть, удовлетворяет ли упорядоченная пара обоим уравнениям.Вскоре мы исследуем способы нахождения такого решения, если оно существует.

\ (\ begin {array} {l} 2 \ left (4 \ right) + \ left (7 \ right) = 15 \ text {} \ text {True} \ hfill \\ 3 \ left (4 \ right) — \ left (7 \ right) = 5 \ text {} \ text {True} \ hfill \ end {array} \)

Помимо учета количества уравнений и переменных, мы можем классифицировать системы линейных уравнений по количеству решений. Согласованная система уравнений имеет по крайней мере одно решение. Согласованной системой считается независимая система , если она имеет единственное решение, такое как пример, который мы только что исследовали.Согласованной системой считается зависимая система , если уравнения представляют разные версии одной и той же линии. Каждая точка на линии представляет пару координат, удовлетворяющую системе. Таким образом, существует бесконечное количество решений.

Другой тип системы линейных уравнений — это несовместимая система . Для обоих уравнений нет общих решений, поэтому нет решения системы.

Как сделать: для данной системы линейных уравнений и упорядоченной пары определите, является ли упорядоченная пара решением.

  1. Подставьте упорядоченную пару в каждое уравнение системы.
  2. Определите, являются ли истинные утверждения результатом подстановки в обоих уравнениях; в таком случае заказанная пара является решением.

Пример

Определите, является ли упорядоченная пара \ (\ left (5,1 \ right) \) решением данной системы уравнений.

\ (\ begin {array} {l} x + 3y = 8 \ hfill \\ 2x — 9 = y \ hfill \ end {array} \)

Показать решение

Подставьте упорядоченную пару \ (\ left (5,1 \ right) \) в оба уравнения.

\ (\ begin {align *} \ left (5 \ right) +3 \ left (1 \ right) & = 8 \\ 8 & = 8 \ hfill & \ text {True} \\ \\ 2 \ left ( 5 \ right) -9 & = \ left (1 \ right) \\ 1 & = 1 \ hfill & \ text {True} \ end {align *} \)

Упорядоченная пара \ (\ left (5,1 \ right) \) удовлетворяет обоим уравнениям, поэтому она является решением системы.

Мы можем ясно увидеть решение, построив график каждого уравнения. Поскольку решение представляет собой упорядоченную пару, удовлетворяющую обоим уравнениям, это точка на обеих прямых и, следовательно, точка пересечения двух прямых.

В следующем видео мы покажем еще один пример того, как проверить, является ли упорядоченная пара решением системы уравнений.

Замена

Существует несколько методов решения систем линейных уравнений. В методе подстановки решения уравнений мы выделяем одну переменную в одном из уравнений, а затем подставляем полученное выражение во второе уравнение, чтобы найти вторую переменную.Поскольку мы можем изолировать только одну переменную за раз, метод подстановки полезен и практичен.

Как сделать: дана система двух уравнений с двумя переменными, решите, используя метод подстановки.

  1. Выберите одну из переменных в одном из уравнений и выделите ее (решите эту переменную через другую переменную).
  2. Подставьте выражение для этой переменной во второе уравнение, затем решите это второе уравнение для оставшейся переменной.
  3. Подставьте это решение в любое из исходных уравнений, чтобы найти значение первой переменной. Если возможно, запишите решение в виде упорядоченной пары.
  4. Проверьте решение в обоих уравнениях.

Пример

Решите следующую систему уравнений путем подстановки.

\ (\ begin {align *} — x + y & = — 5 \\ 2x — 5y & = 1 \ end {align *} \)

Показать решение

Сначала мы решим первое уравнение для \ (y \).

\ (\ begin {align *} — x + y & = — 5 \\ y & = x — 5 \ end {align *} \)

Теперь мы можем заменить выражение \ (x — 5 \) на \ (y \) во втором уравнении.

\ (\ begin {align *} 2x — 5y & = 1 \\ 2x — 5 \ left (x — 5 \ right) & = 1 \\ 2x — 5x + 25 & = 1 \\ -3x & = — 24 \\ x & = 8 \ end {align *} \)

Теперь мы подставляем \ (x = 8 \) в первое уравнение и решаем относительно \ (y \).

\ (\ begin {align *} — \ left (8 \ right) + y & = — 5 \\ y & = 3 \ end {align *} \)

Наше решение — \ (\ left (8,3 \ right) \).

Проверьте решение, подставив \ (\ left (8,3 \ right) \) в оба уравнения.

\ (\ begin {align *} — x + y & = — 5 \\ — \ left (8 \ right) + \ left (3 \ right) & = — 5 \ hfill & \ hfill & \ hfill & \ text {True} \\ \\ 2x — 5y & = 1 \\ 2 \ left (8 \ right) -5 \ left (3 \ right) & = 1 \ hfill & \ hfill & \ hfill & \ text {True} \ конец {выравнивание *} \)

Метод подстановки может использоваться для решения любой линейной системы с двумя переменными, но этот метод работает лучше всего, если одно из уравнений содержит коэффициент 1 или \ (- 1 \), так что нам не нужно иметь дело с дробями.

В следующем видео вам будет показан пример решения системы двух уравнений с использованием метода подстановки.

Если бы вы выбрали другое уравнение для начала в предыдущем примере, вы все равно смогли бы найти такое же решение. Это действительно вопрос предпочтений, потому что иногда решение для переменной приводит к необходимости работать с дробями. По мере того, как вы приобретете больший опыт в алгебре, вы сможете предвидеть, какой выбор приведет к более желаемым результатам.

Напомним, что несовместимая система состоит из параллельных линий, которые имеют одинаковый наклон, но разные точки пересечения y . Они никогда не пересекутся. При поиске решения несовместимой системы мы получим ложное утверждение, например \ (12 = 0 \).

Пример

Решите следующую систему уравнений.

\ (\ begin {align *} x & = 9 — 2y \\ x + 2y & = 13 \ end {align *} \)

Показать решение

Мы можем подойти к этой проблеме двумя способами.Поскольку одно уравнение уже решено для x , наиболее очевидным шагом является использование подстановки.

\ (\ begin {align *} x + 2y & = 13 \\ \ left (9 — 2y \ right) + 2y & = 13 \\ 9 + 0y & = 13 \\ 9 & = 13 \ end {align *} \)

Ясно, что это утверждение противоречит тому, что \ (9 \ ne 13 \). Никакие значения \ (x \) и \ (y \) никогда не сделают \ (9 = 13 \). Следовательно, у системы нет решения.

Второй подход заключается в том, чтобы сначала манипулировать уравнениями, чтобы они оба были в форме пересечения наклона, а затем сравнивать их.Мы манипулируем первым уравнением следующим образом.

\ (\ begin {align *} x & = 9 — 2y \\ 2y & = — x + 9 \\ y & = — \ frac {1} {2} x + \ frac {9} {2} \ end { выровнять *} \)

Затем мы преобразуем второе уравнение в форму с угловым пересечением.

\ (\ begin {align *} x + 2y & = 13 \\ \ 2y & = — x + 13 \\ y & = — \ frac {1} {2} x + \ frac {13} {2} \ end {align *} \)

Сравнивая уравнения, мы видим, что они имеют одинаковый наклон, но разные точки пересечения y . Следовательно, линии параллельны и не пересекаются.Поскольку решения системы являются пересечениями линий, отсутствие пересечений означает, что у системы нет решений.

\ (\ begin {array} {l} \ begin {array} {l} \\ y = — \ frac {1} {2} x + \ frac {9} {2} \ end {array} \ hfill \\ y = — \ frac {1} {2} x + \ frac {13} {2} \ hfill \ end {array} \)

Запись уравнений в форме пересечения наклона подтверждает, что система несовместима, потому что все линии в конечном итоге будут пересекаться, если они не параллельны. Параллельные линии никогда не пересекаются; таким образом, у этих двух линий нет общих точек.Графики уравнений в этом примере показаны ниже.

Ответ

Эта система линейных уравнений не имеет решения.

В следующем видео мы показываем еще один пример использования подстановки для решения системы, у которой нет решения.

В нашем следующем видео мы покажем, что система может иметь бесконечное количество решений.

Рассмотрим функцию дохода производителя скейтбордов , — это функция, используемая для расчета суммы денег, которая поступает в бизнес.Его можно представить уравнением \ (R = xp \), где \ (x = \) количество и \ (p = \) цена. Функция дохода показана оранжевой линией на графике ниже.

Функция затрат — это функция, используемая для расчета затрат на ведение бизнеса. Он включает постоянные затраты, такие как аренда и заработная плата, и переменные затраты, такие как коммунальные услуги. Функция стоимости показана синей линией на графике ниже. Ось x представляет количество скейтбордов, произведенных и проданных в сотнях единиц.Ось y представляет собой стоимость или доход в сотнях долларов.

Точка пересечения двух линий называется точкой безубыточности . Из графика видно, что если \ (x = 7 \) (произведено 700 единиц), то \ (y = 33 \), поэтому стоимость составляет 3300 долларов, а выручка также составляет 3300 долларов. Другими словами, компания сломается, даже если произведет и продаст 700 единиц. Они не зарабатывают и не теряют деньги.

Заштрихованная область справа от точки безубыточности представляет объемы, от которых компания получает прибыль.Заштрихованная область слева представляет объемы, по которым компания терпит убытки.

Пример

Учитывая функцию затрат \ (C \ left (x \ right) = 0,85x + 35,000 \) и функцию дохода \ (R \ left (x \ right) = 1,55x \), найдите точку безубыточности.

Показать решение

Запишите систему уравнений, используя \ (y \) вместо обозначения функций.

\ (\ begin {array} {l} \ begin {array} {l} \\ y = 0,85x + 35,000 \ end {array} \ hfill \\ y = 1,55x \ hfill \ end {array} \)

Подставим выражение \ (0.85x + 35,000 \) из первого уравнения во второе уравнение и решите относительно \ (x \).

\ (\ begin {align *} 0,85x + 35,000 & = 1,55x \\ 35,000 & = 0,7x \\ 50,000 & = x \ end {align *} \)

Затем мы подставляем \ (x = 50,000 \) либо в функцию затрат, либо в функцию дохода.

\ (1,55 \ влево (50,000 \ вправо) = 77,500 \)

Точка безубыточности равна \ (\ left (50,000,77,500 \ right) \).

Стоимость производства 50 000 единиц составляет 77 500 долларов США, а выручка от продажи 50 000 единиц также составляет 77 500 долларов США.Чтобы получить прибыль, бизнес должен произвести и продать более 50 000 единиц.

Компания получит прибыль после выпуска 50 000 единиц.

В следующем примере мы покажем, как написать систему линейных уравнений с учетом данных о посещаемости и стоимости билетов. Затем мы найдем количество купленных билетов на основе нашей системы.

Пример

Стоимость билета в цирк составляет 25 долларов для детей и 50 долларов для взрослых. В определенный день посещаемость цирка составляет 2000 человек, а общий доход от ворот составляет 70 000 долларов.Сколько детей и сколько взрослых купили билеты?

Показать решение

Пусть \ (c = \) количество детей и \ (a = \) количество взрослых, посещающих школу.

Общее количество человек \ (2,000 \). Мы можем использовать это, чтобы написать уравнение количества людей в цирке в этот день

\ (с + а = 2,000 \)

Доход от всех детей можно найти, умножив 25 долларов США на количество детей: \ (25c \). Доход от всех взрослых можно найти, умножив 50 долларов.00 по количеству взрослых: \ (50а \). Общий доход составляет 70 000 долларов. Мы можем использовать это, чтобы написать уравнение дохода.

\ (25c + 50a = 70,000 \)

Теперь у нас есть система линейных уравнений с двумя переменными.

\ (\ begin {array} {c} c + a = 2,000 \\ 25c + 50a = 70,000 \ end {array} \)

Обратите внимание, что каждый член в первом уравнении имеет единицы «люди», а каждый член во втором уравнении имеет единицы «доллары». Создавая систему уравнений из письменного описания, очень важно уделять особое внимание единицам измерения.Помните, что при сложении или вычитании единицы должны совпадать.

В первом уравнении коэффициент при обеих переменных равен 1. Мы можем быстро решить первое уравнение для \ (c \) или \ (a \). Будем решать относительно \ (a \).

\ (\ begin {array} {c} c + a = 2,000 \\ a = 2,000-c \ end {array} \)

Подставьте выражение \ (2,000-c \) во второе уравнение для \ (a \) и решите относительно \ (c \).

\ (\ begin {align *} 25c + 50 \ left (2,000-c \ right) & = 70,000 \\ 25c + 100,000 — 50c & = 70,000 \\ -25c & = — 30,000 \\ \ c & = 1,200 \ конец {выравнивание *} \)

Подставьте \ (c = 1,200 \) в первое уравнение, чтобы найти \ (a \).

\ (\ begin {align *} 1,200 + a & = 2,000 \\ a & = 800 \ end {align *} \)

Мы обнаружили, что \ (1200 \) детей и \ (800 \) взрослых купили билеты в цирк в тот день.

В нашем последнем видео-примере мы показываем, как создать систему линейных уравнений, которая представляет общую стоимость входа в музей.

Ликвидация

Второй метод решения систем линейных уравнений — это метод исключения .В этом методе нет необходимости, но он может быть полезен. Метод замещения часто вводит дроби в процесс решения. Метод исключения не вводит дроби, поэтому некоторые студенты предпочитают исключение замене. Метод исключения имеет тенденцию вводить большие числа в процесс решения, с которыми легче работать, чем с дробями. В методе исключения мы складываем уравнения. Перед этим нам может потребоваться масштабирование одного или обоих уравнений так, чтобы два члена с одной и той же переменной имели равные и противоположные коэффициенты, чтобы их сумма была равна нулю.

Пример

Решите данную систему уравнений методом исключения.

\ (\ begin {align *} x + 2y & = — 1 \\ -x + y & = 3 \ end {align *} \)

Показать решение

Оба уравнения уже установлены равными константе. Обратите внимание, что коэффициент при \ (x \) во втором уравнении, -1, противоположен коэффициенту при \ (x \) в первом уравнении, 1. Мы можем сложить два уравнения, чтобы исключить \ (x \) без необходимости умножать на константу.

\ (\ require {enclose} \ begin {array} {r} x + 2y = -1 \\ \ hfill \ enclose {bottom} {- x + y = 3} \ text {} \ text {} \\ \ hfill 0x + 3y = 2 \ text {} \ text {} \ end {array} \)

Теперь, когда мы исключили \ (x \), мы можем решить полученное уравнение относительно \ (y \).

\ (\ begin {array} {l} 3y = 2 \ hfill \\ \ text {} y = \ frac {2} {3} \ hfill \ end {array} \)

Затем мы подставляем это значение вместо \ (y \) в одно из исходных уравнений и решаем для \ (x \)

\ (\ begin {align *} — x + y & = 3 \\ -x + \ frac {2} {3} & = 3 \\ -x & = 3- \ frac {2} {3} \\ -x & = \ frac {7} {3} \\ x & = — \ frac {7} {3} \ end {align *} \)

Решение этой системы — \ (\ left (- \ frac {7} {3}, \ frac {2} {3} \ right) \).

Проверьте решение в первом уравнении.

\ (\ begin {align *} x + 2y & = — 1 \\ \ left (- \ frac {7} {3} \ right) +2 \ left (\ frac {2} {3} \ right) & = -1 \\ — \ frac {7} {3} + \ frac {4} {3} & = — 1 \\ — \ frac {3} {3} & = — 1 \\ -1 & = — 1 \ hfill & \ hfill & \ hfill & \ text {True} \ hfill \ end {align *} \)

Мы получаем важное представление о системах уравнений, глядя на графическое представление.На графике ниже вы увидите, что уравнения пересекаются в решении. Нам не нужно спрашивать, может ли быть второе решение, потому что наблюдение за графиком подтверждает, что система имеет ровно одно решение.

В следующем видео вы увидите еще один пример того, как использовать метод исключения для решения систем линейных уравнений.

Иногда нам нужно выполнить пару шагов по алгебре, прежде чем мы сможем исключить переменную из системы для ее решения.В следующем примере вы увидите технику, в которой мы умножаем одно из уравнений системы на число, которое позволит нам исключить одну из переменных.

Пример

Решите данную систему уравнений методом исключения .

\ (\ begin {align *} 3x + 5y & = — 11 \\ x — 2y & = 11 \ end {align *} \)

Показать решение

Добавление этих уравнений в представленном виде не устраняет переменную. Мы получили бы \ (4x \) и \ (3y \), и в результате все равно были бы обе переменные.Однако мы видим, что в первом уравнении есть \ (3x \), а во втором уравнении есть \ (x \). Таким образом, если мы масштабируем второе уравнение, умножая его на \ (- 3, \), то у нас будет два уравнения, в которых члены x добавятся к нулю.

\ (\ begin {align *} x — 2y & = 11 \\ -3 \ left (x — 2y \ right) & = — 3 \ left (11 \ right) \ hfill & \ hfill & \ hfill & \ text { Умножьте обе стороны на} -3. \\ -3x + 6y & = — 33 \ hfill & \ hfill & \ hfill & \ text {Используйте свойство распределения}. \\ \ end {align *} \)

Теперь давайте сложим два уравнения вместе: первое уравнение и масштабированную версию второго уравнения.

\ (\ begin {align *} 3x + 5y & = — 11 \\ -3x + 6y & = — 33 \\ \ text {________} & \ text {______} \\ 0x + 11y & = — 44 \\ y & = — 4 \ end {align *} \)

На последнем этапе мы подставляем \ (y = -4 \) в одно из исходных уравнений и решаем относительно \ (x \).

\ (\ begin {align *} 3x + 5y & = — 11 \\ 3x + 5 \ left (-4 \ right) & = — 11 \\ 3x — 20 & = — 11 \\ 3x & = 9 \\ x & = 3 \ end {align *} \)

Наше решение — упорядоченная пара \ (\ left (3, -4 \ right) \). Проверьте решение в исходном втором уравнении.

\ (\ begin {align *} x — 2y & = 11 \\ \ left (3 \ right) -2 \ left (-4 \ right) & = 3 + 8 \\ 11 & = 11 \ hfill & \ hfill & \ hfill & \ text {True} \ hfill \ end {align *} \)

Ниже приведен еще один видео-пример использования метода исключения для решения системы линейных уравнений.

В следующем примере мы увидим, что иногда оба уравнения необходимо умножить на разные числа, чтобы исключить одну переменную.

Пример

Решите данную систему уравнений с двумя переменными методом исключения.

\ (\ begin {align *} 2x + 3y & = — 16 \\ 5x — 10y & = 30 \ end {align *} \)

Показать решение

Одно уравнение имеет \ (2x \), а другое — \ (5x \). Наименьшее общее кратное — это \ (10x \), поэтому нам придется умножить оба уравнения на константу, чтобы исключить одну переменную. Исключим \ (x \), умножив первое уравнение на \ (- 5 \), а второе уравнение на \ (2 \).

\ (\ begin {align *} — 5 \ left (2x + 3y \ right) & = — 5 \ left (-16 \ right) \\ -10x — 15y & = 80 \\ \\ 2 \ left (5x — 10y \ right) & = 2 \ left (30 \ right) \\ 10x — 20y & = 60 \ end {align *} \)

Затем мы складываем два уравнения.

\ (\ begin {align *} — 10x-15y & = 80 \\ 10x-20y & = 60 \\ \ text {_________} & \ text {_______} \\ -35y & = 140 \\ y & = — 4 \ end { выровнять *} \)

Подставить \ (y = -4 \) в исходное первое уравнение.

\ (\ begin {align *} 2x + 3 \ left (-4 \ right) & = — 16 \\ 2x — 12 & = — 16 \\ 2x & = — 4 \\ x & = — 2 \ end {align *} \)

Решение: \ (\ left (-2, -4 \ right) \). Проверьте это в другом уравнении.

\ (\ begin {array} {r} \ hfill \ text {} 5x — 10y = 30 \\ \ hfill 5 \ left (-2 \ right) -10 \ left (-4 \ right) = 30 \\ \ hfill \ text {} -10 + 40 = 30 \\ \ hfill \ text {} 30 = 30 \\ \ end {array} \)

Ниже приводится сводка общих шагов по использованию метода исключения для решения системы уравнений.

Как: решить систему уравнений методом исключения.

  1. Запишите оба уравнения с переменными x и y слева от знака равенства и константами справа.
  2. Напишите одно уравнение над другим, выровняв соответствующие переменные. Если одна из переменных в верхнем уравнении имеет коэффициент, противоположный той же переменной в нижнем уравнении, сложите уравнения вместе, исключив одну переменную.Если нет, используйте умножение на ненулевое число, чтобы одна из переменных в верхнем уравнении имела коэффициент, противоположный той же переменной в нижнем уравнении, затем добавьте уравнения, чтобы исключить переменную.
  3. Решите полученное уравнение для оставшейся переменной.
  4. Подставьте это значение в одно из исходных уравнений и решите для второй переменной.
  5. Проверьте решение, подставив значения в другое уравнение.

В следующем примере мы покажем, как решить систему с дробями.Иногда дроби неизбежны при использовании метода подстановки, но использование метода исключения позволяет очистить их с наименьшим общим знаменателем.

Пример

Решите данную систему уравнений с двумя переменными методом исключения.

\ (\ begin {array} {l} \ frac {x} {3} + \ frac {y} {6} = 3 \ hfill \\ \ frac {x} {2} — \ frac {y} {4 } = \ text {} 1 \ hfill \ end {array} \)

Показать решение

Сначала очистите каждое уравнение от дробей, умножив обе части уравнения на наименьший общий знаменатель этого уравнения

\ (\ begin {align *} 6 \ left (\ frac {x} {3} + \ frac {y} {6} \ right) & = 6 \ left (3 \ right) \\ 2x + y & = 18 \\ \\ 4 \ left (\ frac {x} {2} — \ frac {y} {4} \ right) & = 4 \ left (1 \ right) \\ 2x-y & = 4 \ end {align * } \)

Мы могли бы умножить второе уравнение на \ (- 1 \), чтобы исключить переменную x .Мы также могли бы добавить уравнения напрямую и исключить переменную y . Обе стратегии работают одинаково хорошо. До сих пор мы исключили x , поэтому давайте на этот раз исключим y .

\ (\ begin {align *} 2x + y & = 18 \\ 2x-y & = 4 \\ \ text {________} & \ text {_____} \\ 4x + 0y & = 22 \\ x & = \ frac {11} {2} \ end {align *} \)

Подставьте \ (x = \ frac {11} {2} \) в первое уравнение.

\ (\ begin {align *} 2 \ left (\ frac {11} {2} \ right) + y & = 18 \\ 11 + y & = 18 \\ y & = 7 \ end {align *} \)

Решение: \ (\ left (\ frac {11} {2}, 7 \ right) \).Проверьте это в оригинальном выражении.

\ (\ begin {array} {c} \ frac {x} {2} — \ frac {y} {4} = 1 \\ \ frac {\ frac {11} {2}} {2} — \ frac {7} {4} = 1 \\ \ frac {11} {4} — \ frac {7} {4} = 1 \\ \ frac {4} {4} = 1 \ end {array} \)

В следующем видео вы найдете еще один пример использования метода исключения для решения системы, в котором коэффициенты являются дробными.

Напомним, что зависимая система уравнений с двумя переменными — это система, в которой два уравнения представляют одну и ту же линию.Зависимые системы имеют бесконечное количество решений, потому что все точки на одной линии также находятся на другой линии. После использования замены или исключения результирующее уравнение будет тождественным, например \ (7 = 7 \). Последний пример включает два уравнения, которые представляют одну и ту же линию и, следовательно, являются зависимыми.

Пример

Найдите решение системы уравнений с помощью метода исключения .

\ (\ begin {array} {c} x + 3y = 2 \\ 3x + 9y = 6 \ end {array} \)

Показать решение

Используя метод исключения, мы хотим исключить одну из переменных, добавив уравнения.В этом случае давайте сосредоточимся на устранении \ (x \). Если мы умножим обе части первого уравнения на \ (- 3 \), то мы сможем исключить \ (x \) -переменную.

\ (\ begin {array} {l} \ text {} x + 3y = 2 \ hfill \\ \ left (-3 \ right) \ left (x + 3y \ right) = \ left (-3 \ right) \ left (2 \ right) \ hfill \\ \ text {} -3x — 9y = -6 \ hfill \ end {array} \)

Теперь сложите уравнения.

\ (\ begin {array} \ hfill-3x-9y = -6 \\ \ hfill + 3x + 9y = 6 \\ \ hfill \ text {_____________} \\ \ hfill 0 = 0 \ end {array} \)

Мы видим, что будет бесконечное число решений, удовлетворяющих обоим уравнениям.

Если бы мы переписали оба уравнения в форме пересечения наклона, мы могли бы знать, как будет выглядеть решение перед добавлением. Давайте посмотрим, что происходит, когда мы преобразуем систему в форму с пересечением уклона.

\ (\ begin {array} {l} \ text {} x + 3y = 2 \ hfill \\ \ text {} 3y = -x + 2 \ hfill \\ \ text {} y = — \ frac {1} {3} x + \ frac {2} {3} \ hfill \\ \\ 3x + 9y = 6 \ hfill \\ \ text {} 9y = -3x + 6 \ hfill \\ \ text {} y = — \ frac {3} {9} x + \ frac {6} {9} \ hfill \\ \ text {} y = — \ frac {1} {3} x + \ frac {2} {3} \ hfill \ end {array} \)

См. График ниже.Обратите внимание, что результаты такие же. Общее решение системы — \ (\ left (x, — \ frac {1} {3} x + \ frac {2} {3} \ right) \). Форма решения говорит, что для любого значения x соответствующее значение y будет \ (- \ frac {1} {3} x + \ frac {2} {3} \).

В следующем видео мы показываем еще один пример решения системы, которая зависит от исключения.

В нашем последнем видео-примере мы представляем систему, которая несовместима — у нее нет решений, что означает, что линии, представленные уравнениями, параллельны друг другу.

5.3 Решение систем уравнений методом исключения — элементарная алгебра 2e

Задачи обучения

К концу этого раздела вы сможете:

  • Решите систему уравнений методом исключения
  • Решить приложения систем уравнений методом исключения
  • Выберите наиболее удобный метод решения системы линейных уравнений

Будьте готовы 5,8

Прежде чем начать, пройдите тест на готовность.

Упростим −5 (6−3a) −5 (6−3a).
Если вы пропустили эту проблему, просмотрите Пример 1.136.

Будьте готовы 5.9

Решите уравнение 13x + 58 = 312413x + 58 = 3124.
Если вы пропустили эту проблему, просмотрите Пример 2.48.

Мы решили системы линейных уравнений с помощью построения графиков и подстановки. Построение графиков хорошо работает, когда переменные коэффициенты малы, а решение имеет целочисленные значения. Подстановка работает хорошо, когда мы можем легко решить одно уравнение для одной из переменных и не иметь слишком много дробей в результирующем выражении.

Третий метод решения систем линейных уравнений называется методом исключения. Когда мы решали систему с помощью подстановки, мы начинали с двух уравнений и двух переменных и сводили ее к одному уравнению с одной переменной. То же самое мы сделаем и с методом исключения, но у нас будет другой способ добиться этого.

Решите систему уравнений методом исключения

Метод исключения основан на добавочном свойстве равенства. Свойство сложения равенства говорит, что когда вы добавляете одинаковую величину к обеим сторонам уравнения, вы все равно получаете равенство.Мы расширим свойство сложения равенства, чтобы сказать, что когда вы добавляете равные количества к обеим сторонам уравнения, результаты равны.

Для любых выражений a , b , c и d ,

ifa = bandc = dthena + c = b + difa = bandc = dthena + c = b + d

Чтобы решить систему уравнений методом исключения, мы начнем с обоих уравнений в стандартной форме. Затем мы решаем, какую переменную будет легче всего устранить. Как мы решаем? Мы хотим, чтобы коэффициенты одной переменной были противоположными, чтобы мы могли сложить уравнения и исключить эту переменную.

Обратите внимание, как это работает, когда мы складываем эти два уравнения вместе:

3x + y = 52x − y = 0 _________ 5x = 53x + y = 52x − y = 0 _________ 5x = 5

и прибавляют к нулю, и мы получаем одно уравнение с одной переменной.

Давайте попробуем еще один:

{x + 4y = 22x + 5y = −2 {x + 4y = 22x + 5y = −2

На этот раз мы не видим переменную, которую можно было бы немедленно удалить, если мы добавим уравнения.

Но если мы умножим первое уравнение на −2, мы сделаем коэффициенты при x противоположными.Мы должны умножить каждый член в обеих частях уравнения на −2.

Теперь мы видим, что коэффициенты членов x противоположны, поэтому x будет исключено, когда мы сложим эти два уравнения.

Сложите уравнения самостоятельно — результат должен быть −3 y = −6. И это кажется несложным, не так ли? Вот как бы это выглядело.

Сделаем еще один:

{4x − 3y = 103x + 5y = −7 {4x − 3y = 103x + 5y = −7

Не похоже, чтобы коэффициенты одной переменной были противоположными, умножая одно из уравнений на константу, если мы не используем дроби.Поэтому вместо этого нам придется умножить оба уравнения на константу.

Мы можем сделать коэффициенты x противоположными, если умножим первое уравнение на 3, а второе на −4, так что мы получим 12 x и −12 x .

Это дает нам два новых уравнения:

{12x − 9y = 30−12x − 20y = 28 {12x − 9y = 30−12x − 20y = 28

Когда мы складываем эти уравнения,

{12x − 9y = 30−12x − 20y = 28 _____________− 29y = 58 {12x − 9y = 30−12x − 20y = 28 _____________− 29y = 58

исключаются x , и остается −29 y = 58.

Как только мы получаем уравнение с одной переменной, мы его решаем. Затем мы подставляем это значение в одно из исходных уравнений, чтобы найти оставшуюся переменную. И, как всегда, мы проверяем наш ответ, чтобы убедиться, что он является решением обоих исходных уравнений.

Теперь мы увидим, как использовать исключение для решения той же системы уравнений, которую мы решили с помощью построения графиков и подстановки.

Пример 5.25

Как решить систему уравнений методом исключения

Решите систему устранением.{2x + y = 7x − 2y = 6 {2x + y = 7x − 2y = 6

Попробовать 5,49

Решите систему устранением. {3x + y = 52x − 3y = 7 {3x + y = 52x − 3y = 7

Попробовать 5.50

Решите систему устранением. {4x + y = −5−2x − 2y = −2 {4x + y = −5−2x − 2y = −2

Шаги перечислены ниже для удобства.

How To

Как решить систему уравнений методом исключения.
  1. Шаг 1. Запишите оба уравнения в стандартной форме. Если какие-либо коэффициенты являются дробными, очистите их.
  2. Шаг 2. Сделайте коэффициенты одной переменной противоположными.
    • Решите, какую переменную исключить.
    • Умножьте одно или оба уравнения так, чтобы коэффициенты этой переменной были противоположными.
  3. Шаг 3. Добавьте уравнения, полученные на шаге 2, чтобы исключить одну переменную.
  4. Шаг 4. Найдите оставшуюся переменную.
  5. Шаг 5. Подставьте решение из шага 4 в одно из исходных уравнений. Затем найдите другую переменную.
  6. Шаг 6. Запишите решение в виде упорядоченной пары.
  7. Шаг 7. Убедитесь, что упорядоченная пара является решением обоих исходных уравнений.

Сначала мы приведем пример, в котором мы можем сразу исключить одну переменную.

Пример 5.26

Решите систему устранением. {x + y = 10x − y = 12 {x + y = 10x − y = 12

Попробуйте 5.51

Решите систему устранением. {2x + y = 5x − y = 4 {2x + y = 5x − y = 4

Попробуйте 5.52

Решите систему устранением.{x + y = 3−2x − y = −1 {x + y = 3−2x − y = −1

В примере 5.27 мы сможем сделать коэффициенты одной переменной противоположными, умножив одно уравнение на константу.

Пример 5.27

Решите систему устранением. {3x − 2y = −25x − 6y = 10 {3x − 2y = −25x − 6y = 10

Попробовать 5.53

Решите систему устранением. {4x − 3y = 15x − 9y = −4 {4x − 3y = 15x − 9y = −4

Попробовать 5.54

Решите систему устранением. {3x + 2y = 26x + 5y = 8 {3x + 2y = 26x + 5y = 8

Теперь мы рассмотрим пример, в котором нам нужно умножить оба уравнения на константы, чтобы сделать коэффициенты одной переменной противоположными.

Пример 5.28

Решите систему устранением. {4x − 3y = 97x + 2y = −6 {4x − 3y = 97x + 2y = −6

Решение

В этом примере мы не можем умножить одно уравнение на любую константу, чтобы получить противоположные коэффициенты. Поэтому мы стратегически умножим оба уравнения на константу, чтобы получить противоположности.

Какие еще константы мы могли бы выбрать для исключения одной из переменных? Будет ли решение таким же?

Попробовать 5.55

Решите систему устранением. {3x − 4y = −95x + 3y = 14 {3x − 4y = −95x + 3y = 14

Попробовать 5.56

Решите систему устранением. {7x + 8y = 43x − 5y = 27 {7x + 8y = 43x − 5y = 27

Когда система уравнений содержит дроби, мы сначала очистим дроби, умножив каждое уравнение на его ЖК-дисплей.

Пример 5.29

Решите систему устранением. {x + 12y = 632x + 23y = 172 {x + 12y = 632x + 23y = 172

Решение

В этом примере в обоих уравнениях есть дроби.Нашим первым шагом будет умножение каждого уравнения на его ЖК-дисплей, чтобы очистить дроби.

Попробовать 5.57

Решите систему устранением. {13x − 12y = 134x − y = 52 {13x − 12y = 134x − y = 52

Попробовать 5.58

Решите систему устранением. {x + 35y = −15−12x − 23y = 56 {x + 35y = −15−12x − 23y = 56

В разделе «Решение систем уравнений с помощью графического представления» мы увидели, что не все системы линейных уравнений имеют единственную упорядоченную пару в качестве решения. Когда два уравнения действительно представляли собой одну и ту же линию, решений было бесконечно много.Мы назвали это последовательной системой. Когда два уравнения описывали параллельные линии, решения не было. Мы назвали это несовместимой системой.

Пример 5.30

Решите систему устранением. {3x + 4y = 12y = 3−34x {3x + 4y = 12y = 3−34x

Решение
{3x + 4y = 12y = 3−34x {3x + 4y = 12y = 3−34x
Запишите второе уравнение в стандартной форме. {3x + 4y = 1234x + y = 3 {3x + 4y = 1234x + y = 3
Очистите дроби, умножив второе уравнение на 4. {3x + 4y = 124 (34x + y) = 4 (3) {3x + 4y = 124 (34x + y) = 4 (3)
Упростить. {3x + 4y = 123x + 4y = 12 {3x + 4y = 123x + 4y = 12
Чтобы исключить переменную, мы умножаем второе уравнение на -1-1.
Упростите и добавьте.
{3x + 4y = 12−3x − 4y = −12 ________________ 0 = 0 {3x + 4y = 12−3x − 4y = −12 ________________ 0 = 0

Это верное заявление. Уравнения непротиворечивы, но зависимы. Их графики будут одной линией.У системы бесконечно много решений.

После того, как мы очистили дроби во втором уравнении, заметили ли вы, что эти два уравнения совпадают? Это означает, что у нас есть совпадающие линии.

Попробовать 5.59

Решите систему устранением. {5x − 3y = 15y = −5 + 53x {5x − 3y = 15y = −5 + 53x

Попробовать 5.60

Решите систему устранением. {x + 2y = 6y = −12x + 3 {x + 2y = 6y = −12x + 3

Пример 5.31

Решите систему устранением.{−6x + 15y = 102x − 5y = −5 {−6x + 15y = 102x − 5y = −5

Решение
Уравнения имеют стандартную форму. {−6x + 15y = 102x − 5y = −5 {−6x + 15y = 102x − 5y = −5
Умножьте второе уравнение на 3, чтобы исключить переменную. {−6x + 15y = 103 (2x − 5y) = 3 (−5) {- 6x + 15y = 103 (2x − 5y) = 3 (−5)
Упростить и добавить. {−6x + 15y = 106x − 15y = −15 __________________ 0 ≠ −5 {−6x + 15y = 106x − 15y = −15 __________________ 0 ≠ −5

Это утверждение неверно.Уравнения несовместимы, поэтому их графики будут параллельными линиями.

В системе нет решения.

Попробовать 5.61

Решите систему устранением. {−3x + 2y = 89x − 6y = 13 {−3x + 2y = 89x − 6y = 13

Попробовать 5.62

Решите систему устранением. {7x − 3y = −2−14x + 6y = 8 {7x − 3y = −2−14x + 6y = 8

Решение приложений систем уравнений методом исключения

Некоторые прикладные задачи переводятся непосредственно в уравнения стандартной формы, поэтому мы будем использовать метод исключения для их решения.Как и прежде, мы используем нашу стратегию решения проблем, чтобы оставаться сосредоточенными и организованными.

Пример 5.32

Сумма двух чисел равна 39. Их разница равна 9. Найдите числа.

Решение
Шаг 1. Прочтите проблему.
Шаг 2. Определите то, что мы ищем. Ищем два числа.
Шаг 3.Назовите то, что мы ищем.
Выберите переменную для представления этого количества.
Пусть n = n = первое число.
м = м = второе число.
Шаг 4. Переведите в систему уравнений.



Система:
Сумма двух чисел равна 39.
n + m = 39n + m = 39
Их разница равна 9.
n − m = 9 {n + m = 39n − m = 9n − m = 9 {n + m = 39n − m = 9
Шаг 5. Решите систему уравнений.
Для решения системы уравнений используйте исключение.
Уравнения имеют стандартную форму, а коэффициенты при мм противоположны. Добавлять.


Решите для nn.


Подставьте n = 24n = 24 в одно из исходных уравнений и решите относительно мм.
{n + m = 39n − m = 9 ____________ 2n = 48n = 24n + m = 3924 + m = 39m = 15 {n + m = 39n − m = 9 ____________ 2n = 48n = 24n + m = 3924 + m = 39m = 15
Шаг 6. Отметьте ответ. Поскольку 24 + 15 = 3924 + 15 = 39 и 24-15 = 924-15 = 9, ответы проверяются.
Шаг 7. Ответьте на вопрос. Цифры 24 и 15.

Попробовать 5.63

Сумма двух чисел равна 42. Их разность равна 8. Найдите числа.

Попробовать 5.64

Сумма двух чисел равна −15. Их разница -35. Найдите числа.

Пример 5.33

Джо каждый день останавливается в ресторане, где подают гамбургеры, по дороге на работу. В понедельник он заказал средний картофель фри и два небольших газированных напитка, в которых в общей сложности было 620 калорий.Во вторник у него было два заказа среднего картофеля фри и одна небольшая газировка, в общей сложности 820 калорий. Сколько калорий в одной порции среднего картофеля фри? Сколько калорий в одной маленькой газировке?

Попробовать 5.65

Малик останавливается в продуктовом магазине, чтобы купить пакет подгузников и 2 банки смеси. В общей сложности он тратит 37 долларов. На следующей неделе он останавливается и покупает 2 пакета подгузников и 5 банок смеси на общую сумму 87 долларов. Сколько стоит сумка с подгузниками? Сколько можно смеси?

Попробовать 5.66

Чтобы получить ежедневное количество фруктов в течение дня, Саша съела банан и 8 ягод клубники в среду, получив 145 калорий. В следующую среду она съела два банана и 5 ягод клубники, в общей сложности 235 калорий для фруктов. Сколько калорий в банане? Сколько калорий в клубнике?

Выберите наиболее удобный метод решения системы линейных уравнений

Когда вам придется решать систему линейных уравнений в более позднем математическом классе, вам обычно не сообщают, какой метод использовать.Вам нужно будет принять это решение самостоятельно. Так что вы захотите выбрать самый простой метод, который сводит к минимуму ваши шансы на ошибку.

Пример 5.34

Для каждой системы линейных уравнений решите, что удобнее будет решить заменой или исключением. Поясните свой ответ.

ⓐ {3x + 8y = 407x − 4y = −32 {3x + 8y = 407x − 4y = −32 ⓑ {5x + 6y = 12y = 23x − 1 {5x + 6y = 12y = 23x − 1

Решение
  1. ⓐ {3x + 8y = 407x − 4y = −32 {3x + 8y = 407x − 4y = −32
    Поскольку оба уравнения имеют стандартную форму, использование исключения будет наиболее удобным.
  2. ⓑ {5x + 6y = 12y = 23x − 1 {5x + 6y = 12y = 23x − 1

> Поскольку одно уравнение уже решено для y , использование подстановки будет наиболее удобным.

Попробовать 5.67

Решите для каждой системы линейных уравнений, что удобнее решить: заменой или исключением. Поясните свой ответ.

ⓐ {4x − 5y = −323x + 2y = −1 {4x − 5y = −323x + 2y = −1 ⓑ {x = 2y − 13x − 5y = −7 {x = 2y − 13x − 5y = −7

Попробовать 5.68

Решите для каждой системы линейных уравнений, что удобнее решить: заменой или исключением.Поясните свой ответ.

ⓐ {y = 2x − 13x − 4y = −6 {y = 2x − 13x − 4y = −6 ⓑ {6x − 2y = 123x + 7y = −13 {6x − 2y = 123x + 7y = −13

Раздел 5.3. Упражнения

Практика ведет к совершенству

Решите систему уравнений методом исключения

В следующих упражнениях решите системы уравнений методом исключения.

126.

{5x + 2y = 2−3x − y = 0 {5x + 2y = 2−3x − y = 0

127.

{−3x + y = −9x − 2y = −12 {−3x + y = −9x − 2y = −12

128.

{6x − 5y = −12x + y = 13 {6x − 5y = −12x + y = 13

129.

{3x − y = −74x + 2y = −6 {3x − y = −74x + 2y = −6

130.

{x + y = −1x − y = −5 {x + y = −1x − y = −5

131.

{x + y = −8x − y = −6 {x + y = −8x − y = −6

132.

{3x − 2y = 1 − x + 2y = 9 {3x − 2y = 1 − x + 2y = 9

133.

{−7x + 6y = −10x − 6y = 22 {−7x + 6y = −10x − 6y = 22

134.

{3x + 2y = −3 − x − 2y = −19 {3x + 2y = −3 − x − 2y = −19

135.

{5x + 2y = 1−5x − 4y = −7 {5x + 2y = 1−5x − 4y = −7

136.

{6x + 4y = −4−6x − 5y = 8 {6x + 4y = −4−6x − 5y = 8

137.

{3x − 4y = −11x − 2y = −5 {3x − 4y = −11x − 2y = −5

138.

{5x − 7y = 29x + 3y = −3 {5x − 7y = 29x + 3y = −3

139.

{6x − 5y = −75 − x − 2y = −13 {6x − 5y = −75 − x − 2y = −13

140.

{−x + 4y = 83x + 5y = 10 {−x + 4y = 83x + 5y = 10

141.

{2x − 5y = 73x − y = 17 {2x − 5y = 73x − y = 17

142.

{5x − 3y = −12x − y = 2 {5x − 3y = −12x − y = 2

143.

{7x + y = −413x + 3y = 4 {7x + y = −413x + 3y = 4

144.

{−3x + 5y = −132x + y = −26 {−3x + 5y = −132x + y = −26

145.

{3x − 5y = −95x + 2y = 16 {3x − 5y = −95x + 2y = 16

146.

{4x − 3y = 32x + 5y = −31 {4x − 3y = 32x + 5y = −31

147.

{4x + 7y = 14−2x + 3y = 32 {4x + 7y = 14−2x + 3y = 32

148.

{5x + 2y = 217x − 4y = 9 {5x + 2y = 217x − 4y = 9

149.

{3x + 8y = −32x + 5y = −3 {3x + 8y = −32x + 5y = −3

150.

{11x + 9y = −57x + 5y = −1 {11x + 9y = −57x + 5y = −1

151.

{3x + 8y = 675x + 3y = 60 {3x + 8y = 675x + 3y = 60

152.

{2x + 9y = −43x + 13y = −7 {2x + 9y = −43x + 13y = −7

153.

{13x − y = −3x + 52y = 2 {13x − y = −3x + 52y = 2

154.

{x + 12y = 3215x − 15y = 3 {x + 12y = 3215x − 15y = 3

155.

{x + 13y = −112x − 13y = −2 {x + 13y = −112x − 13y = −2

156.

{13x − y = −323x + 52y = 3 {13x − y = −323x + 52y = 3

157.

{2x + y = 36x + 3y = 9 {2x + y = 36x + 3y = 9

158.

{x − 4y = −1−3x + 12y = 3 {x − 4y = −1−3x + 12y = 3

159.

{−3x − y = 86x + 2y = −16 {−3x − y = 86x + 2y = −16

160.

{4x + 3y = 220x + 15y = 10 {4x + 3y = 220x + 15y = 10

161.

{3x + 2y = 6−6x − 4y = −12 {3x + 2y = 6−6x − 4y = −12

162.

{5x − 8y = 1210x − 16y = 20 {5x − 8y = 1210x − 16y = 20

163.

{−11x + 12y = 60−22x + 24y = 90 {−11x + 12y = 60−22x + 24y = 90

164.

{7x − 9y = 16−21x + 27y = −24 {7x − 9y = 16−21x + 27y = −24

165.

{5x − 3y = 15y = 53x − 2 {5x − 3y = 15y = 53x − 2

166.

{2x + 4y = 7y = −12x − 4 {2x + 4y = 7y = −12x − 4

Решение приложений систем уравнений методом исключения

В следующих упражнениях переведите в систему уравнений и решите.

167.

Сумма двух чисел равна 65. Их разница равна 25. Найдите числа.

168.

Сумма двух чисел равна 37. Их разница равна 9. Найдите числа.

169.

Сумма двух чисел равна −27. Их разница -59. Найдите числа.

170.

Сумма двух чисел равна -45. Их разница -89. Найдите числа.

171.

Андреа покупает новые рубашки и свитера. Она может купить 3 рубашки и 2 свитера за 114 долларов или 2 рубашки и 4 свитера за 164 доллара. Сколько стоит рубашка? Сколько стоит свитер?

172.

Питер покупает канцелярские товары. Он может купить 3 пакета бумаги и 4 степлера за 40 долларов, или он может купить 5 пакетов бумаги и 6 степлеров за 62 доллара. Сколько стоит пачка бумаги? Сколько стоит степлер?

173.

Общее количество натрия в 2 хот-догах и 3 чашках творога составляет 4720 мг. Общее количество натрия в 5 хот-догах и 2 стаканах творога составляет 6300 мг. Сколько натрия в хот-доге? Сколько натрия в стакане творога?

174.

Общее количество калорий в 2 хот-догах и 3 чашках творога составляет 960 калорий. Общее количество калорий в 5 хот-догах и 2 стаканах творога составляет 1190 калорий. Сколько калорий в хот-доге? Сколько калорий в стакане творога?

Выберите наиболее удобный метод решения системы линейных уравнений

В следующих упражнениях решите, что было бы удобнее решить систему уравнений путем подстановки или исключения.

175.


ⓐ {8x − 15y = −326x + 3y = −5 {8x − 15y = −326x + 3y = −5 ⓑ {x = 4y − 34x − 2y = −6 {x = 4y − 34x − 2y = −6

176.


ⓐ {y = 7x − 53x − 2y = 16 {y = 7x − 53x − 2y = 16 ⓑ {12x − 5y = −423x + 7y = −15 {12x − 5y = −423x + 7y = −15

177.


ⓐ {y = 4x + 95x − 2y = −21 {y = 4x + 95x − 2y = −21 ⓑ {9x − 4y = 243x + 5y = −14 {9x − 4y = 243x + 5y = −14

178.


ⓐ {14x − 15y = −307x + 2y = 10 {14x − 15y = −307x + 2y = 10 ⓑ {x = 9y − 112x − 7y = −27 {x = 9y − 112x − 7y = −27

Повседневная математика
179.

Норрис может греб на 3 мили против течения за 1 час, столько же времени у него уходит на греблю на 5 миль вниз по течению.Решите систему. {r − c = 3r + c = 5 {r − c = 3r + c = 5

  1. ⓐ для rr, его скорость гребли в стоячей воде.
  2. ⓑ Затем решите относительно cc, скорость течения реки.
180.

Джози хочет приготовить 10 фунтов смеси для троп из орехов и изюма, и она хочет, чтобы общая стоимость смеси для трейлов составила 54 доллара. Орехи стоят 6 долларов за фунт, а изюм — 3 доллара за фунт. Решите систему {n + r = 106n + 3r = 54 {n + r = 106n + 3r = 54, чтобы найти nn, количество фунтов орехов, и rr, количество фунтов изюма, которое она должна использовать.

Письменные упражнения
181.

Решите систему
{x + y = 105x + 8y = 56 {x + y = 105x + 8y = 56

ⓐ путем замены Ⓑ путем построения графиков Ⓒ Какой метод вы предпочитаете? Почему?

182.

Решите систему
{x + y = −12y = 4−12x {x + y = −12y = 4−12x

ⓐ путем замены Ⓑ путем построения графиков Ⓒ Какой метод вы предпочитаете? Почему?

Самопроверка

ⓐ После выполнения упражнений используйте этот контрольный список, чтобы оценить свое мастерство в достижении целей этого раздела.

ⓑ Что этот контрольный список говорит вам о вашем мастерстве в этом разделе? Какие шаги вы предпримете для улучшения?

.

Добавить комментарий

Ваш адрес email не будет опубликован. Обязательные поля помечены *